Preview (15 of 70 pages)

ATI Pediatrics
1. A nurse is assessing a school-aged child who has heart failure and is taking furosemide. Which
of the following findings should the nurse identify as an indication that the medication is
effective?
A. Decreased blood pressure
B. Increased urine output
C. Decreased respiratory rate
D. Weight gain
Answer: B. Increased urine output
Rationale:
Furosemide is a loop diuretic used to manage fluid overload in heart failure. An effective
response to furosemide therapy is indicated by an increase in urine output, as the medication
promotes diuresis (increased urine production). Other options, such as decreased blood pressure
and decreased respiratory rate, may occur but are not direct indicators of the effectiveness of
furosemide. Weight gain would indicate fluid retention, which would not be a desired outcome
when using furosemide.
2. A nurse is assessing an infant who has acute otitis media. Which of the following findings
should the nurse expect? (Select all that apply)
A. Crying
B. Restlessness
C. Fever
D. Rashes
Answer: A. Crying
B. Restlessness
C. Fever
Rationale:
In infants with acute otitis media, common findings include:
• Crying (A): Infants may be irritable or cry more than usual due to ear pain.
• Restlessness (B): Discomfort from the infection can lead to restlessness and difficulty sleeping.

• Fever (C): A mild to moderate fever is often present as part of the body's response to infection.
• Rashes (D) are not a typical finding associated with acute otitis media and may indicate another
condition, so it is not included as a correct answer.
3. A nurse is providing teaching to the parents of an infant who is to undergo pilocarpine
iontophoresis testing for cystic fibrosis. Which of the following statements should the nurse
include in the teaching?
A. The test will measure the amount of sodium in your baby’s sweat.
B. The test will measure the amount of water in your baby’s sweat.
C. The test will determine if your baby has an allergy to pilocarpine.
D. The test will identify the presence of lung infections.
Answer: B. The test will measure the amount of water in your baby’s sweat.
Rationale:
• Pilocarpine iontophoresis testing, also known as the sweat test, is used to diagnose cystic
fibrosis by measuring the concentration of chloride in the sweat. The process stimulates sweat
production, and the resulting sweat is collected for analysis.
• The statement about measuring water content is correct in that the test indirectly assesses sweat
composition, which involves electrolytes like sodium and chloride.
• While sodium is measured, the key aspect of the test focuses on chloride levels, making option
B the most accurate response in this context.
4. A nurse in an urgent care clinic is prioritizing care for children. Which of the following
children should the nurse assess first?
A. A preschool-aged child who has a muffled voice and no spontaneous cough
B. A school-aged child with a fever of 101°F and a sore throat
C. An infant with a runny nose and a mild cough
D. A toddler who is experiencing diarrhoea and vomiting
Answer: A. A preschool-aged child who has a muffled voice and no spontaneous cough
Rationale:
• The preschool-aged child with a muffled voice and no spontaneous cough is exhibiting signs
that may indicate a serious condition, such as epiglottitis or severe airway obstruction. The

muffled voice suggests potential swelling in the throat, and the lack of a cough raises concern for
airway compromise.
• In comparison:
• The school-aged child with fever and sore throat may need assessment but is not as urgent as
airway issues.
• The infant with a runny nose and mild cough is likely dealing with a viral upper respiratory
infection, which is typically less severe.
• The toddler with diarrhoea and vomiting needs attention, but this does not present the
immediate risk to airway that the child with a muffled voice does.
• Thus, assessing the preschool-aged child first is critical to prevent possible airway compromise.
5. A nurse is providing teaching to the parents of a toddler who is to undergo a sweat chloride
test. Which of the following statements should the nurse include?
A. Your child's sweat will be collected over 24 hours.
B. The test will measure the amount of chloride in your child's urine.
C. Your child will have a small electrical current applied to stimulate sweating.
D. Your child should avoid drinking fluids for 12 hours before the test.
Answer: C. Your child will have a small electrical current applied to stimulate sweating.
Rationale:
The sweat chloride test is used primarily to diagnose cystic fibrosis. During the procedure, a
small electrical current is applied to the skin using a device called an iontophoresis machine,
which stimulates the sweat glands to produce sweat. The collected sweat is then analyzed for
chloride content.
6. A nurse in the emergency department is caring for an adolescent who is requesting testing for
an STI. Which of the following actions is appropriate for the nurse to take?
A. Obtain written consent from the client.
B. Inform the client that parental consent is required for testing.
C. Schedule a follow-up appointment for testing at a later date.
D. Provide education on safe sex practices before testing.
Answer: D. Provide education on safe sex practices before testing.
Rationale:

When an adolescent requests STI testing, it is crucial to support their autonomy and provide
necessary education. Educating the client on safe sex practices can help prevent future infections
and promote overall sexual health. In many jurisdictions, adolescents have the legal right to
consent to STI testing without parental involvement, depending on their age and local laws.
7. A nurse in the emergency department is assessing a toddler who has hyperpyrexia, severe
dyspnea, and drooling. Which of the following actions should the nurse take first?
A. Prepare the toddler for nasotracheal intubation.
B. Administer antipyretics to reduce fever.
C. Assess the toddler’s airway and respiratory status.
D. Provide humidified oxygen to the toddler.
Answer: A. Prepare the toddler for nasotracheal intubation.
Rationale:
In an emergency situation involving a toddler with hyperpyrexia, severe dyspnea, and drooling,
the priority action is to assess the airway and respiratory status. Severe dyspnea and drooling can
indicate potential airway obstruction or significant respiratory distress, which requires immediate
evaluation to ensure the toddler can breathe adequately.
8. A nurse is providing teaching to a 10-year-old child scheduled for an arterial cardiac
catheterization. Which of the following information should the nurse include in the teaching?
A. You will be required to drink plenty of fluids after the procedure.
B. You will need to keep your legs straight for 8 hours following the procedure.
C. You will be allowed to eat immediately after the procedure.
D. You will receive a sedative to help you relax during the procedure.
Answer: B. You will need to keep your legs straight for 8 hours following the procedure.
Rationale:
Keeping the legs straight for a specified duration after an arterial cardiac catheterization is crucial
to prevent complications such as bleeding or hematoma formation at the catheter insertion site.
9. A nurse is caring for a preschooler who is post-operative following a tonsillectomy. The child is
now ready to resume oral intake. Which of the following dietary choices should the nurse offer
the child?

A. Lime flavored ice pop
B. Orange juice
C. Chocolate milkshake
D. Spicy chicken broth
Answer: A. Lime flavored ice pop
Rationale:
After a tonsillectomy, it is important to offer soft, soothing, and non-irritating foods and fluids to
prevent discomfort and promote healing.
10. A nurse is caring for an infant who has patent ductus arteriosus (PDA). The nurse should
identify that the defect is a switch of the following locations of the heart.
A. Aorta
B. Ductus arteriosus
C. Pulmonary artery
D. Left atrium
Answer: B. Ductus arteriosus
Rationale:
• Patent ductus arteriosus is a condition where the ductus arteriosus, a blood vessel connecting the
aorta and pulmonary artery, fails to close after birth. This results in abnormal blood flow between
these two vessels.
• Option A (aorta) is not correct because the aorta itself is not switched but is involved in the
abnormal flow.
• Option B (ductus arteriosus) is correct as the PDA is specifically related to the ductus arteriosus
remaining open.
• Option C (pulmonary artery) is also not correct because the pulmonary artery is involved in the
condition but is not the site of the defect.
• Option D (left atrium) is incorrect since the left atrium is not involved in the PDA.
Thus, the defect specifically involves the ductus arteriosus remaining patent, making option B the
correct choice.
11. A nurse is caring for a 10• month• old child brought to the emergency department by his
parents following a head injury. Which of the following actions should the nurse take first?

A. Assess respiratory status
B. Obtain a history from the parents
C. Perform a neurological assessment
D. Check the child’s vital signs
Answer: A. Assess respiratory status
Rationale:
• In emergency situations, especially with head injuries, ensuring the child's airway and
respiratory status is the top priority. Any compromise in breathing could lead to severe
complications, so the nurse should first assess respiratory status to ensure the child is stable.
• Option B (obtain a history from the parents) is important but secondary to immediate
physiological assessments.
• Option C (perform a neurological assessment) is also crucial but should follow the initial
assessment of respiratory status.
• Option D (check the child’s vital signs) is necessary but can be included in the overall
assessment after ensuring the airway is clear and breathing is adequate.
• Therefore, the correct first action is to assess respiratory status.
12. A charge nurse is planning care for an infant who has failure to thrive. Which of the following
actions should the nurse include in the plan of care?
A. Assign the consistent nursing staff to care for the infant
B. Allow the parents to visit only during scheduled hours
C. Encourage the infant to feed independently
D. Limit the number of caregivers involved with the infant
Answer: A. Assign the consistent nursing staff to care for the infant
Rationale:
• Consistency in nursing care is crucial for infants with failure to thrive. Assigning the same
nursing staff helps build trust and rapport, which can lead to better assessments of the infant's
needs and responses to interventions.
• Option B (allow the parents to visit only during scheduled hours) is not supportive; parents
should be encouraged to spend time with their infant.
• Option C (encourage the infant to feed independently) may not be appropriate, as infants with
failure to thrive often require assistance and support during feeding.

• Option D (limit the number of caregivers involved with the infant) is generally good practice;
however, the focus should be on maintaining a consistent nursing staff rather than simply limiting
caregivers.
• Therefore, the best action is to assign consistent nursing staff to provide care for the infant.
13. A nurse is providing teaching about home care to the parent of a child who has scabies. Which
of the following instructions should the nurse include in the teaching?
A. Treat everyone who came into close contact with the child
B. Allow the child to return to school after 24 hours of treatment
C. Wash all bedding and clothing in cold water
D. Apply the topical treatment only to the affected areas
Answer: A. Treat everyone who came into close contact with the child
Rationale:
• Scabies is highly contagious and can easily spread through close physical contact. It is essential
to treat not only the infected child but also all individuals who have had close contact with them
to prevent reinfestation and further spread of the infestation.
• Option B (allow the child to return to school after 24 hours of treatment) is incorrect; typically,
children should stay home until they have been treated and are free of symptoms.
• Option C (wash all bedding and clothing in cold water) is not sufficient; items should be washed
in hot water to effectively kill the scabies mites.
• Option D (apply the topical treatment only to the affected areas) is not recommended; the
treatment should be applied to the entire body from the neck down to ensure that any mites are
eradicated.
• Thus, the most important instruction is to treat everyone who came into close contact with the
child.
14. A nurse is caring for a preschooler who refuses to take a dose of oral diphenhydramine.
Which of the following statements should the nurse make?
A. Sometimes, when children have to take medication, they feel sad.
B. If you don’t take your medicine, you will not feel better.
C. Let’s make a game out of taking your medicine.
D. This medicine will help you sleep better tonight.

Answer: C. Let’s make a game out of taking your medicine.
Rationale:
• Using play or making a game out of taking medication can help to reduce anxiety and resistance
in preschoolers. Engaging the child in a fun activity can distract them and make the experience
more positive.
• Option A (sometimes, when children have to take medication, they feel sad) may acknowledge
the child’s feelings but does not provide a solution or encouragement.
• Option B (if you don’t take your medicine, you will not feel better) can create fear or anxiety
about their health but does not help in the moment.
• Option D (this medicine will help you sleep better tonight) could be misleading if the
medication is not specifically for sleep and does not address the child's refusal.
• Therefore, making it a game is a more effective and supportive approach for encouraging
medication adherence in a preschooler.
15. A nurse is teaching the parent of a school age child bicycle safety. Which of the following
instructions should the nurse include in the teaching?
A. Your child should ride on the sidewalk at all times.
B. Your child should walk the bicycle through intersections.
C. Your child should wear a helmet only during long rides.
D. Your child can ride against traffic to see oncoming vehicles.
Answer: B. Your child should walk the bicycle through intersections.
Rationale:
• Walking the bicycle through intersections helps ensure the child is more aware of their
surroundings and reduces the risk of accidents. Intersections can be particularly dangerous for
cyclists due to the potential for unseen vehicles and the need for quick decision making.
• Option A (your child should ride on the sidewalk at all times) may not always be appropriate, as
some areas may require riding in the street.
• Option C (your child should wear a helmet only during long rides) is incorrect; helmets should
be worn every time the child rides, regardless of distance.
• Option D (your child can ride against traffic to see oncoming vehicles) is unsafe; children
should always ride with traffic to follow the flow and be more predictable to drivers.

• Emphasizing safe behaviors like walking the bicycle through intersections contributes to overall
safety.
16. A nurse is caring for a school age child following the application of a cast to a fractured right
tibia. Which of the following actions should the nurse take first?
A. Pad the edges of the cast.
B. Assess the child's neurovascular status.
C. Teach the child about cast care.
D. Elevate the affected leg.
Answer: B. Assess the child's neurovascular status.
Rationale:
• The priority action following the application of a cast is to assess the child's neurovascular
status. This assessment helps ensure that there is adequate circulation and nerve function in the
limb, which is crucial to detect any complications such as compartment syndrome or impaired
blood flow early on.
• Option A (pad the edges of the cast) is important for comfort and to prevent skin irritation, but it
is not the immediate priority.
• Option C (teach the child about cast care) is also important but should come after ensuring that
there are no immediate neurovascular issues.
• Option D (elevate the affected leg) is important to reduce swelling but should be done after
assessing the child's neurovascular status.
• Prioritizing neurovascular assessment helps in early detection and intervention for potential
complications.
17. A nurse is preparing a school aged child for an invasive procedure. Which of the following
actions should the nurse plan to take?
A. Plan for a 30 minute teaching session about the procedure.
B. Allow the child to choose the site for the procedure.
C. Schedule the procedure without explaining it to the child.
D. Use complex medical terminology to explain the procedure.
Answer: A. Plan for a 30 minute teaching session about the procedure.
Rationale:

• Planning a teaching session about the procedure is crucial for helping the child understand what
to expect. This age group is typically able to comprehend basic explanations and can ask
questions. Providing clear, age appropriate information can help reduce anxiety and foster
cooperation.
• Option B (allow the child to choose the site) is not typically appropriate for invasive procedures,
as the site is determined by medical necessity.
• Option C (schedule the procedure without explaining it) can increase anxiety and fear in the
child, making it essential to provide an explanation beforehand.
• Option D (use complex medical terminology) is not suitable, as it may confuse the child rather
than help them understand the procedure.
• Effective communication tailored to the child's developmental level is key in preparing them for
an invasive procedure.
18. A nurse is preparing to collect a urine specimen from a female infant using a urine collection
bag. Which of the following actions should the nurse take?
A. Apply lidocaine gel to the perineum before attaching the bag.
B. Cleanse the perineum with soap and water before attaching the bag.
C. Ensure the collection bag is securely adhered to the skin.
D. Use a cotton ball to absorb any excess moisture before applying the bag.
Answer: C. Ensure the collection bag is securely adhered to the skin.
Rationale:
• When using a urine collection bag for an infant, it is crucial to ensure that the bag is securely
adhered to the skin to prevent leakage and ensure accurate specimen collection. This will help
maintain a proper seal and collect a valid urine sample.
• Option A (apply lidocaine gel) is not typically necessary or recommended for urine collection
using a bag and may cause unnecessary discomfort.
• Option B (cleanse the perineum with soap and water) is not recommended immediately before
applying the bag; gentle cleansing with a sterile wipe is usually sufficient.
• Option D (use a cotton ball to absorb moisture) is not necessary as it could interfere with the
adherence of the collection bag.
• By focusing on securely attaching the collection bag, the nurse can effectively gather a reliable
urine specimen.

19. A nurse is planning care for a toddler who has developed oral ulcers in response to
chemotherapy. Which of the following actions should the nurse include in the plan of care?
A. Schedule routine oral care every 8 hours.
B. Use alcohol based mouthwash for oral hygiene.
C. Encourage the child to eat spicy foods to stimulate appetite.
D. Offer cold or soft foods to minimize pain during eating.
Answer: D. Offer cold or soft foods to minimize pain during eating.
Rationale:
• For a toddler with oral ulcers due to chemotherapy, it's important to provide care that minimizes
pain and promotes healing. Offering cold or soft foods can soothe oral ulcers and make eating
more comfortable for the child.
• Option A (schedule routine oral care every 8 hours) is important, but it may not be frequent
enough; oral care should ideally be done more frequently (at least every 4 hours).
• Option B (use alcohol• based mouthwash) is not recommended, as alcohol can irritate oral
ulcers and exacerbate pain.
• Option C (encourage spicy foods) can cause additional irritation to the ulcers and should be
avoided.
• By including options that prioritize comfort, the nurse can help the toddler manage the side
effects of chemotherapy more effectively.
20. A nurse is providing discharge teaching to the parents of an infant who is at risk for sudden
infant death syndrome (SIDS). Which of the following statements by the parent indicates an
understanding of the teaching?
A. I will dress my baby in lightweight clothing to sleep.
B. I will place my baby on their stomach to sleep.
C. I will keep the baby's crib free of soft toys and blankets.
D. I will place my baby in a shared bed with me for easier monitoring.
Answer: A. I will dress my baby in lightweight clothing to sleep.
Rationale:

• Dressing the baby in lightweight clothing is appropriate to prevent overheating, which is a risk
factor for SIDS. Parents should ensure the infant sleeps in a safe environment, which includes
using a firm mattress and keeping the crib free of soft bedding and toys.
• Option B (placing the baby on their stomach) is not recommended, as infants should be placed
on their backs to sleep to reduce the risk of SIDS.
• Option C (keeping the crib free of soft toys and blankets) is also correct but not the statement
chosen; it reinforces safe sleep practices.
• Option D (placing the baby in a shared bed) is discouraged because it increases the risk of
suffocation and other hazards.
• The correct understanding focuses on reducing risk factors associated with SIDS, including
appropriate clothing and safe sleep positioning.
21. A nurse is monitoring an infant who is receiving opioids for pain. Which of the following
findings should indicate to the nurse that the medication is having a therapeutic effect?
A. Relaxed facial expression
B. Increased heart rate
C. Excessive crying
D. Increased muscle tone
Answer: A. Relaxed facial expression
Rationale:
• A relaxed facial expression in the infant is a key indicator that the opioid medication is
effectively managing pain. Opioids are intended to relieve pain and provide comfort, and a
relaxed demeanour typically reflects a reduction in pain levels.
• Option B (increased heart rate) can be a sign of pain or discomfort, indicating that the
medication may not be effective.
• Option C (excessive crying) is often a response to pain, suggesting that the opioid may not be
providing adequate relief.
• Option D (increased muscle tone) may indicate tension or discomfort, which would not be a
sign of effective pain management.
• Therefore, a relaxed facial expression is the most appropriate indicator of therapeutic effect
from opioid administration in an infant.

22. A nurse is caring for a three month old infant who has a cleft of the soft palate. Which of the
following actions should the nurse take?
A. Discontinue feeding if the client's eyes become watery
B. Use a standard nipple for feeding
C. Encourage the infant to drink from a straw
D. Position the infant in an upright position during feeding
Answer: D. Position the infant in an upright position during feeding
Rationale:
• For an infant with a cleft soft palate, positioning the infant in an upright position during feeding
helps reduce the risk of aspiration and facilitates better swallowing. This position can also help
minimize the flow of milk into the nasal cavity, which is a common issue for infants with cleft
palate.
• Option A (discontinue feeding if the client's eyes become watery) is not an appropriate action, as
watery eyes could be related to various factors and do not necessarily indicate the need to stop
feeding.
• Option B (use a standard nipple for feeding) may not be suitable, as specialized nipples or
feeding devices are often recommended for infants with cleft palate to ensure proper feeding.
• Option C (encourage the infant to drink from a straw) is not appropriate for a three month old
infant, as they typically do not have the developmental ability to use a straw effectively.
• Thus, positioning the infant upright is the best practice to ensure safe and effective feeding.
23. A nurse is caring for a child who has hyponatremia. Which of the following findings should
the nurse expect?
A. Hypertension
B. Decreased urine output
C. Tetany
D. Increased thirst
Answer: C. Tetany
Rationale:
• Hyponatremia (low sodium levels in the blood) can lead to various neurological symptoms due
to the effects on cellular fluid balance. One of the expected findings is tetany, which refers to
muscle spasms or cramps that occur due to increased neuromuscular excitability.

• Option A (hypertension) is not typical of hyponatremia; rather, hyponatremia may lead to
hypotension due to fluid shifts.
• Option B (decreased urine output) is not necessarily indicative of hyponatremia; urine output
can vary depending on the cause of the sodium imbalance.
• Option D (increased thirst) may not be a direct symptom of hyponatremia; instead, thirst may be
more common in hypernatremia (high sodium levels).
• Therefore, tetany is the most appropriate finding to expect in a child with hyponatremia.
24. A nurse is preparing to administer an enteral feeding to an adolescent who has an NG tube.
Which of the following actions should the nurse take first?
A. Check the pH of the gastric secretion
B. Assess the placement of the NG tube
C. Warm the feeding solution
D. Administer the feeding via gravity
Answer: B. Assess the placement of the NG tube
Rationale:
• Before administering enteral feedings, it is crucial to ensure that the NG tube is properly
positioned in the stomach to prevent complications such as aspiration or feeding into the lungs.
While checking the pH of gastric secretions (Option A) is an important step to confirm
placement, the initial action should be to assess the placement of the tube directly, which may
include checking for residuals or auscultating for air entry.
• Option A (check the pH of the gastric secretion) is a subsequent step after verifying placement.
• Option C (warm the feeding solution) is important for patient comfort but comes after
confirming tube placement.
• Option D (administer the feeding via gravity) should only be done after confirming that the NG
tube is correctly positioned.
• Thus, the priority action is to assess the placement of the NG tube first.
25. A nurse is caring for an adolescent who is 1 hour post operative following an appendectomy.
Which of the following findings should the nurse report to the provider?
A. Mild abdominal tenderness
B. Muscle rigidity

C. Low grade fever
D. Drowsiness
Answer: B. Muscle rigidity
Rationale:
• Muscle rigidity in a post operative patient, especially following an appendectomy, can indicate
potential complications such as peritonitis or internal bleeding. This finding is concerning and
should be reported to the provider immediately for further evaluation.
• Option A (mild abdominal tenderness) can be expected post operatively and is not usually
alarming unless severe.
• Option C (low grade fever) can also be a normal response to surgery but should be monitored.
• Option D (drowsiness) can be a side effect of anaesthesia and may not be concerning as long as
the patient is arousable and responsive.
• Thus, muscle rigidity is a significant finding that requires immediate attention.
26. A nurse in a provider’s office is preparing to administer immunization to a 12-year-old client
during a well-child visit. Which of the following immunizations should the nurse plan to
administer?
A. Human papillomavirus (HPV)
B. Meningococcal conjugate vaccine (MCV4)
C. Diphtheria, tetanus, and pertussis (DTaP) booster
D. Hepatitis B vaccine
Answer: A. Human papillomavirus (HPV)
Rationale:
• The Human papillomavirus (HPV) vaccine is recommended for preteens (ages 11-12) to protect
against cancers caused by HPV, including cervical cancer and some other genital cancers. It is
typically administered as a two- or three-dose series, starting at this age.
• Option B (Meningococcal conjugate vaccine) is also recommended around this age, but the
HPV vaccine is specifically highlighted in this context.
• Option C (DTaP booster) is not typically given at this age; adolescents usually receive the Tdap
(tetanus, diphtheria, and pertussis) booster instead.
• Option D (Hepatitis B vaccine) should have been completed in infancy and early childhood.
• Given the importance of HPV vaccination for this age group, the correct choice is A.

27. A nurse is planning care for an 8-month-old infant who has heart failure. Which of the
following actions should the nurse include in the plan of care?
A. Provide less frequent, higher volume feeding
B. Allow the infant to feed in a supine position
C. Use a soft nipple with a large opening for feeding
D. Offer small, frequent feedings
Answer: D. Offer small, frequent feedings
Rationale:
• In infants with heart failure, it’s important to minimize the work of feeding and prevent fatigue.
Offering small, frequent feedings helps ensure that the infant receives adequate nutrition without
overexertion. This approach can help manage the symptoms of heart failure and support the
infant's growth and development.
• Option A (provide less frequent, higher volume feeding) could lead to fatigue and decreased
intake.
• Option B (allow the infant to feed in a supine position) is not recommended as it can increase
the risk of aspiration.
• Option C (use a soft nipple with a large opening) might lead to choking or aspiration, as it can
allow too much formula to flow quickly.
• Thus, offering small, frequent feedings is the best strategy to support the infant's nutritional
needs while managing heart failure.
28. A nurse is planning care for a school-aged child who is admitted from the emergency
department 12 hours ago. Which of the following interventions should the nurse include to
promote adequate sleep for the child?
A. Allow the child to adjust their bedtime to promote autonomy
B. Provide a quiet, dark environment for sleeping
C. Encourage the child to engage in stimulating activities before bedtime
D. Limit fluid intake to prevent nighttime awakenings
Answer: B. Provide a quiet, dark environment for sleeping
Rationale:

• Creating a quiet, dark environment is essential for promoting restful sleep, especially in a
hospital setting where noises and lights can disrupt sleep patterns. This intervention helps the
child feel more comfortable and allows for better sleep quality.
• Option A (allowing the child to adjust their bedtime) may not be appropriate in a hospital
setting, as it can lead to irregular sleep patterns.
• Option C (encouraging stimulating activities) can hinder the child’s ability to fall asleep and
should be avoided before bedtime.
• Option D (limiting fluid intake) may prevent nighttime awakenings but can also lead to
dehydration and is not necessary for promoting sleep.
• Thus, providing a quiet, dark environment is the most effective way to support the child's need
for adequate sleep.
29. A nurse is preparing to initiate IV antibiotic therapy for a newly admitted 12-month-old
infant. Which of the following actions should the nurse plan to take?
A. Use a 24 gauge catheter to start the IV
B. Select the antecubital vein for insertion
C. Use a 22 gauge catheter for insertion
D. Apply a warm compress to the insertion site
Answer: A. Use a 24 gauge catheter to start the IV
Rationale:
• In infants, a 24 gauge catheter is appropriate for IV access due to their smaller veins and the
need to minimize trauma. It provides a good balance between ensuring adequate flow for
medication delivery while reducing the risk of complications such as infiltration or phlebitis.
• Option B (selecting the antecubital vein) is generally not preferred for infants due to the risk of
complications and the difficulty of securing the IV.
• Option C (using a 22 gauge catheter) may be too large for a 12-month-old's veins, increasing the
risk of trauma.
• Option D (applying a warm compress) can help dilate veins but is not necessary as the primary
action for initiating IV therapy.
• Thus, using a 24 gauge catheter is the best choice for initiating IV antibiotic therapy in this age
group.

30. A nurse in a paediatric clinic is providing teaching to the guardian of an infant who has a new
prescription for digoxin. Which of the following manifestations should the nurse include as an
indication of digoxin toxicity?
A. Bradycardia
B. Increased appetite
C. Hyperactivity
D. Diarrhoea
Answer: A. Bradycardia
Rationale:
• Bradycardia is a significant indicator of digoxin toxicity, especially in infants, where the normal
heart rate is typically higher than in adults. Other signs of digoxin toxicity may include nausea,
vomiting, visual disturbances, and arrhythmias. It is important for guardians to monitor for these
signs, as digoxin has a narrow therapeutic window and can easily lead to toxicity.
• Option B (increased appetite) is not associated with digoxin toxicity; in fact, loss of appetite
may be a sign of toxicity.
• Option C (hyperactivity) is also not a common symptom of digoxin toxicity; lethargy or
decreased activity may be more indicative.
• Option D (diarrhoea) can occur but is not as specific as bradycardia for indicating digoxin
toxicity.
• Therefore, bradycardia is the correct manifestation to include in teaching about digoxin toxicity.
31. A nurse is reviewing the laboratory results of a child who was recently admitted for suspected
rheumatic fever. The nurse should identify that which of the following laboratory tests can
contribute to confirm this diagnosis? (Select all that apply)
A. Erythrocyte sedimentation rate (ESR)
B. C-reactive protein (CRP)
C. Anti-streptolysin O (ASO) titter
D. Diarrhoea
Answer: • A. Erythrocyte sedimentation rate (ESR)
• B. C-reactive protein (CRP)
• C. Anti-streptolysin O (ASO) titer
Rationale:

• All three tests can help support the diagnosis of rheumatic fever:
• Erythrocyte sedimentation rate (ESR): This test measures inflammation in the body and is often
elevated in cases of rheumatic fever.
• C-reactive protein (CRP): Similar to ESR, CRP is an indicator of inflammation and can be
elevated in rheumatic fever.
• Anti-streptolysin O (ASO) titter: This test specifically checks for antibodies against streptolysin
O, a toxin produced by group A Streptococcus, which is the bacteria that can cause rheumatic
fever. An elevated ASO titter suggests a recent streptococcal infection.
• Thus, all three tests are relevant for confirming the diagnosis of rheumatic fever.
32. A nurse is teaching a group of female adolescents about healthy eating. Which of the
following instructions should the nurse include in the teaching?
A. Increase the amount of your dietary iron intake
B. Decrease your intake of fruits and vegetables
C. Limit your protein sources to only dairy products
D. Avoid all carbohydrates in your diet
Answer: Increase the amount of your dietary iron intake.
Rationale:
Adolescents, especially females, are at a higher risk for iron deficiency due to factors such as
menstruation, which can lead to increased iron loss. Adequate iron intake is crucial for preventing
anaemia and supporting overall health, particularly as they grow and develop. The nurse should
encourage them to consume iron-rich foods such as lean meats, beans, fortified cereals, and leafy
green vegetables, along with vitamin C sources to enhance iron absorption.
33. A nurse is caring for an infant who receives intermittent enteral feeding through a gastrostomy
tube. Which of the following actions should the nurse take when administering a feeding? (Select
all that apply)
A. Offer an infant a pacifier during feedings
B. Check for residual volume by aspirating stomach contents
C. In still the formula over a period of 30 to 45 minutes
D. Position the infant on their back during feedings
E. Warm the formula to room temperature before administration

Answer: A. Offer an infant a pacifier during feedings
B. Check for residual volume by aspirating stomach contents
C. In still the formula over a period of 30 to 45 minutes
Rationale:
• A. Offer an infant a pacifier during feedings: This can help soothe the infant and promote a
sense of normalcy during feedings.
• B. Check for residual volume by aspirating stomach contents: This is essential to assess gastric
emptying and determine if it's safe to proceed with the feeding.
• C. In still the formula over a period of 30 to 45 minutes: Administering the formula slowly helps
prevent gastrointestinal distress and ensures better tolerance.
34. A nurse is planning care for a child who has osteomyelitis. Which of the following
interventions should the nurse include in the plan of care? Maintain patent intravenous catheter
A. Maintain patent intravenous catheter
B. Apply heat to the affected area
C. Encourage weight-bearing activities
D. Provide a high-protein diet
Answer: A. Maintain patent intravenous catheter
Rationale:
• A. Maintain patent intravenous catheter: Children with osteomyelitis often require prolonged
intravenous antibiotic therapy, making it essential to maintain a patent IV line for medication
administration.
• B. Apply heat to the affected area: This is generally not recommended as it can increase
swelling and promote infection.
• C. Encourage weight-bearing activities: This is contraindicated during the acute phase of
osteomyelitis due to the risk of worsening the condition.
• D. Provide a high-protein diet: While nutritional support is important, it is not a primary
intervention specifically targeted at managing osteomyelitis. The priority remains on managing
the infection and ensuring effective antibiotic therapy.
35. A nurse is providing teaching to a guardian of a school-aged child who has sickle cell disease
about management of the illness. Which of the following instructions should the nurse include?

A. Encourage physical activity as tolerated
B. Limit fluid intake to avoid frequent urination
C. Administer aspirin for pain relief as needed
D. Avoid exposure to extreme temperatures
Answer: A. Encourage physical activity as tolerated
Rationale:
• A. Encourage physical activity as tolerated: This is important for maintaining overall health and
promoting blood circulation. However, it's crucial to ensure that the child does not overexert
themselves, especially during a sickle cell crisis.
• B. Limit fluid intake to avoid frequent urination: This is incorrect; adequate hydration is
essential in managing sickle cell disease to help prevent sickle cell crises.
• C. Administer aspirin for pain relief as needed: While aspirin may be used for some pain
management, it is not typically recommended for children with sickle cell disease due to the risk
of complications like Reye's syndrome. Instead, acetaminophen or other pain management
strategies may be advised.
• D. Avoid exposure to extreme temperatures: While this is important for overall health, the best
practice is to encourage a balanced approach to activities and environmental factors rather than
avoiding them altogether.
• Thus, the most appropriate instruction to include in teaching is to encourage physical activity as
tolerated.
36. A nurse is assessing a 5 month old infant. Which of the following findings should the nurse
report to the provider?
A. Exhibits head lag when pulled to a sitting position
B. Has a social smile
C. Babbles when excited
D. Grasps objects with both hands
Answer: A. Exhibits head lag when pulled to a sitting position
Rationale:
• A. Exhibits head lag when pulled to a sitting position: This is concerning because by 5 months,
infants should be able to maintain head control and have minimal head lag when pulled to a

sitting position. Persistent head lag may indicate developmental delays or neurological concerns
that need further evaluation.
• B. Has a social smile: This is a normal finding for a 5-month-old and indicates social
engagement.
• C. Babbles when excited: Babbling is typical behavior at this age and shows developing
communication skills.
• D. Grasps objects with both hands: This is a normal developmental milestone at this age,
showing improving motor skills.
• Thus, the finding that should be reported is the head lag when pulled to a sitting position, as it
could indicate a potential developmental issue.
37. A nurse is caring for a five-year-old child following a tonsillectomy and adenoidectomy.
Which of the following findings should the nurse identify as an indication of haemorrhage?
A. Continuous swallowing
B. Low-grade fever
C. Complaints of throat pain
D. Increased appetite
Answer: A. Continuous swallowing
Rationale:
• A. Continuous swallowing: This is a key indicator of potential haemorrhage after a
tonsillectomy and adenoidectomy. Continuous swallowing may indicate that the child is trying to
clear blood from the throat or is experiencing bleeding, which requires immediate assessment and
intervention.
• B. Low-grade fever: This can be a normal postoperative finding and is not necessarily indicative
of haemorrhage.
• C. Complaints of throat pain: While pain is common after the procedure, it is not a specific sign
of haemorrhage.
• D. Increased appetite: This is generally not expected after surgery, as children often have
reduced appetite due to discomfort. An increased appetite does not suggest haemorrhage.
• Thus, continuous swallowing is the finding that should raise concern for possible haemorrhage
following the procedure.

38. A nurse is discussing coping mechanism with a parent of a three-month-old infant which of
the following therapeutic questions should the nurse ask the parent?
A. Is it overwhelming when your infant is having a bad day?
B. Are you keeping up with all the baby’s needs?
C. Do you find it easy to care for your infant?
D. How often do you feel supported by others?
Answer: A. Is it overwhelming when your infant is having a bad day?
Rationale:
• A. Is it overwhelming when your infant is having a bad day?: This question is therapeutic as it
allows the parent to express their feelings about the challenges of parenting an infant, particularly
during difficult times. It encourages open communication about stressors and coping
mechanisms.
• B. Are you keeping up with all the baby’s needs?: While this question addresses the parent’s
ability to manage care, it may not encourage emotional expression as effectively as option A.
• C. Do you find it easy to care for your infant?: This question may dismiss the struggles parents
often face, which can lead to feelings of inadequacy.
• D. How often do you feel supported by others?: While this question is relevant, it is more
focused on external support rather than the immediate emotional response to challenging
situations with the infant.
• Thus, option A effectively prompts the parent to reflect on their feelings and coping strategies
during stressful moments.
39. A nurse is providing teaching about the effects of sun exposure to a parent of a toddler which
of the following responses by the parent indicates an understanding of the teaching?
A. I should apply 10 SPF sunscreen to my child’s entire body.
B. I can skip sunscreen on cloudy days.
C. My child can wear a swim shirt and not need sunscreen.
D. I will reapply sunscreen every 2 hours, especially after swimming.
Answer: D. I will reapply sunscreen every 2 hours, especially after swimming.
Rationale:

• A. I should apply 10 SPF sunscreen to my child’s entire body: This indicates some
understanding but is inadequate. The American Academy of Dermatology recommends using a
broad• spectrum sunscreen with at least SPF 30 for effective protection.
• B. I can skip sunscreen on cloudy days: This statement shows a misunderstanding. UV rays can
penetrate clouds, so sunscreen is still necessary on cloudy days.
• C. My child can wear a swim shirt and not need sunscreen: While swim shirts provide some
protection, they do not eliminate the need for sunscreen, especially on exposed skin.
• D. I will reapply sunscreen every 2 hours, especially after swimming: This response shows a
correct understanding of proper sun protection practices, emphasizing the importance of
reapplication after swimming or sweating, which is essential for effective sun safety.
• Thus, option D reflects a comprehensive understanding of sun protection for toddlers.
40. A nurse is evaluating a 6 year old who has cystic fibrosis and has been receiving chest
physiotherapy treatment. The nurse should identify which of the following findings as an
indication of the therapy has been effective?
A. Decreased heart rate
B. Increased expectoration
C. Improved oxygen saturation
D. Decreased respiratory effort
Answer: B. Increased expectoration
Rationale:
• In cystic fibrosis, chest physiotherapy is designed to help clear mucus from the lungs. An
effective treatment would be indicated by an increase in expectoration, meaning the child is able
to cough up and expel more mucus. This suggests that the therapy is helping to clear airway
obstruction, improving lung function.
• A. Decreased heart rate: While this could indicate improvement, it is not a direct measure of the
effectiveness of chest physiotherapy.
• C. Improved oxygen saturation: This is important but may not be immediately measurable as a
direct outcome of chest physiotherapy alone.
• D. Decreased respiratory effort: This could suggest improved breathing, but again, the most
direct indicator of effective chest physiotherapy in cystic fibrosis is the increased expectoration of
mucus.

• Thus, option B is the most appropriate indicator of effective therapy.
41. A nurse is planning care for a six month old infant who has bacterial meningitis. Which of the
following interventions should the nurse include in the plan of care?
A. Keep the room well lit to promote stimulation
B. Pad the side rails of the crib
C. Encourage oral feeding to maintain hydration
D. Place the infant in a prone position
Answer: B. Pad the side rails of the crib
Rationale:
• Padding the side rails of the crib is a necessary intervention for a six month old infant with
bacterial meningitis, as the condition can increase the risk of seizures. Padding helps prevent
injury if the infant has a seizure or experiences sudden movements.
• A. Keep the room well lit to promote stimulation: This is not appropriate for a child with
meningitis, as excessive stimulation can be harmful.
• C. Encourage oral feeding to maintain hydration: While hydration is important, infants with
meningitis may require IV fluids instead, especially if they are lethargic or unable to feed.
• D. Place the infant in a prone position: This is not recommended for an infant with meningitis; a
supine or side lying position is safer to reduce the risk of aspiration and allow for easier
breathing.
• Thus, option B is the most appropriate intervention to include in the care plan for the infant.
42. A nurse is reviewing the medical record of a child with cystic fibrosis which of the following
should the nurse report to the provider? Click on the exhibit button for addition information about
the client.
A. HbA1c of 5.0%
B. HbA1c of 7.2%
C. HbA1c of 6.0%
D. HbA1c of 8.5%
Answer: D. HbA1c of 8.5%
Rationale:

An HbA1c level of 8.5% indicates poor glycaemic control and suggests that the child may have
diabetes, which is a common complication in individuals with cystic fibrosis. This warrants
further assessment and management, and should be reported to the provider.
43. A nurse is assessing an infant who has severe dehydration due to gastroenteritis which of the
following findings should the nurse expect?
A. Decreased respiratory rate
B. Increased respiratory rate
C. Normal respiratory rate
D. Periodic breathing
Answer: B. Increased respiratory rate
Rationale:
In cases of severe dehydration, the body may compensate for decreased blood volume and
pressure by increasing the respiratory rate. This response helps to improve oxygenation and is
often seen in infants experiencing dehydration due to conditions like gastroenteritis. Other signs
of severe dehydration may include lethargy, dry mucous membranes, and decreased urine output.
44. A nurse is assessing an infant who has intussusception. Which of the following findings
should the nurse expect?
A. Sausage-shaped abdominal mass
B. Rigid abdomen
C. Absent bowel sounds
D. Projectile vomiting
Answer: A. Sausage-shaped abdominal mass
Rationale:
Intussusception in infants often presents with a classic "sausage-shaped" abdominal mass,
typically felt in the upper abdomen. This condition occurs when a part of the intestine telescopes
into an adjacent segment, leading to obstruction. Other common signs may include intermittent
abdominal pain, "currant jelly" stools, and signs of distress, but the sausage-shaped mass is a key
characteristic finding during assessment.

45. A nurse is caring for a 14-year-old adolescent who has a cast on the right arm and swelling of
their right hand. The nurse elevates the adolescents affected extremity The nurse should identify
that which of the following findings is an indication that the intervention has been effective?
A. The adolescent is able to move their fingers freely
B. The skin over the cast is warm to the touch
C. There is increased swelling in the hand
D. The adolescent reports less pain in the arm
Answer: A. The adolescent is able to move their fingers freely
Rationale:
Elevating the affected extremity is intended to reduce swelling and promote circulation. If the
adolescent is able to move their fingers freely, it indicates that circulation has improved and that
there is likely a reduction in swelling or pressure within the cast. This finding suggests that the
intervention was effective. Other options, such as increased swelling or warmth of the skin over
the cast, could indicate complications like compartment syndrome, which would be concerning.
46. A nurse in a provider’s office is assessing the vital signs of a two-year-old child at a well-child
visit. Which of the following findings should the nurse report to the provider?
A. Heart rate 110 beats per minute
B. Blood pressure 118/74 mm Hg
C. Respiratory rate 20 breaths per minute
D. Temperature 99.5°F (37.5°C)
Answer: B. Blood pressure 118/74 mm Hg
Rationale:
For a two-year-old child, a blood pressure reading of 118/74 mm Hg is significantly elevated and
may indicate hypertension. Normal blood pressure for this age group typically ranges from about
90/50 mm Hg to 110/70 mm Hg, depending on size and growth. The other vital signs provided
(heart rate, respiratory rate, temperature) are within normal limits for a toddler. Therefore, the
elevated blood pressure should be reported to the provider for further evaluation.
47. A nurse is preparing to administer a prescribed medication to a toddler whose parent is
nearby. Which of the following actions should the nurse take to identify the toddler?
A. Ask the parent to confirm the toddler's name

B. Check the toddler’s ID band against the medical record
C. Verify the toddler's name with another nurse
D. Use the toddler’s room number for identification
Answer: B. Check the toddler’s ID band against the medical record
Rationale:
The most reliable method for identifying a patient, especially a toddler, is to check their ID band
against the medical record. This ensures that the correct medication is administered to the right
child. While asking the parent for confirmation can be helpful, it is not as secure as verifying the
ID band, which is a standard protocol to prevent medication errors. Using a room number is also
not a reliable identification method.
48. A nurse is teaching home care to the parents of a pre-schooled-age child who has heart failure.
Which of the following information should the nurse include in the teaching?
A. Increase physical activity to improve strength
B. Provide for periods of rest
C. Limit fluid intake significantly
D. Monitor for weight loss
Answer: B. Provide for periods of rest
Rationale:
Children with heart failure often experience fatigue and decreased exercise tolerance. Providing
for periods of rest is essential to help manage their energy levels and prevent exacerbation of
symptoms. While monitoring fluid intake is important, it should be done according to the
healthcare provider's recommendations, and encouraging physical activity should be balanced
with the child's ability and condition. Weight loss is typically not a goal for children with heart
failure; rather, maintaining a healthy weight is important.
49. A nurse in the PACU is caring for a school-aged child immediately following a tonsillectomy.
Which of the following actions should the nurse make?
A. Position the child in a supine position.
B. Place the child in a side-lying position.
C. Elevate the child's head to a 90-degree angle.
D. Keep the child in a prone position.

Answer: B. Place the child in a side-lying position.
Rationale:
After a tonsillectomy, placing the child in a side-lying position helps to facilitate drainage of
secretions and reduces the risk of aspiration. This position can also provide comfort and minimize
the risk of airway obstruction. The child should not be placed supine immediately after surgery,
as this could increase the risk of aspiration if there is any bleeding or pooling of secretions.
50. A nurse is reviewing the medical record of a 15-month-old child who is scheduled to receive
measles, mumps, rubella. Which of the following findings should the nurse identify as a
contradiction for receiving the vaccine?
A. History of mild illness
B. Allergy to eggs
C. Allergy to neomycin
D. Recent immunization with varicella
Answer: C. Allergy to neomycin.
Rationale:
The measles, mumps, rubella (MMR) vaccine contains neomycin, which is an antibiotic. A
known allergy to neomycin is a contraindication for receiving the MMR vaccine because it may
lead to an allergic reaction. Other options, such as a history of mild illness or recent
immunizations with varicella, are not typically contraindications for administering the MMR
vaccine.
51. A nurse is assessing a school-aged child cranial nerve function. Which of the following
actions should the nurse ask the child to take when assessing the accessory nerve?
A. Stick out their tongue
B. Smile and show their teeth
C. Shrug their shoulders against mild pressure
D. Close their eyes tightly
Answer: C. Shrug their shoulders against mild pressure.
Rationale:
The accessory nerve (cranial nerve XI) is responsible for the movement of the
sternocleidomastoid and trapezius muscles. To assess this nerve, the nurse should ask the child to

shrug their shoulders against mild pressure. This action tests the strength of the trapezius muscle,
indicating proper function of the accessory nerve. The other options assess different cranial
nerves.
52. A nurse is performing a cranial nerve assessment on a school-age child. Which of the
following findings indicates proper function of the child trigeminal nerve?
A. The child has asymmetrical jaw strength when biting down
B. The child can feel light touch on the forehead
C. The child can close their eyes tightly
D. The child can smile and show their teeth
Answer: B. The child can feel light touch on the forehead.
Rationale:
The trigeminal nerve (cranial nerve V) is responsible for sensation in the face and motor
functions such as biting and chewing. Proper function is indicated by the child being able to feel
light touch on the forehead, which tests the sensory component of the trigeminal nerve.
Asymmetrical jaw strength when biting down (Option A) suggests a potential issue with nerve
function and is not a sign of proper function. Options C and D assess different cranial nerves
(facial nerve and oculomotor nerve, respectively).
53. A nurse is providing support to a family whose infant died from sudden infant death syndrome
(SIDS) which of the following actions should the nurse take?
A. Acknowledge the family members' feelings of guilt.
B. Encourage the family to move on quickly.
C. Tell the family that SIDS is common and they are not alone.
D. Suggest the family should not talk about the infant to avoid pain.
Answer: A. Acknowledge the family members' feelings of guilt.
Rationale:
Acknowledging feelings of guilt is important as it validates the family's emotions. Many parents
experience guilt after the loss of a child, and recognizing this feeling can help them process their
grief and begin healing.

54. A nurse in the emergency department is caring for a child who has a temperature of 39.1C
(102.4F) and suspected the diagnosis of bacterial meningitis. Which of the following actions
should the nurse take first?
A. Dim the lights in the child’s room
B. Obtain a complete blood count (CBC)
C. Administer an antipyretic medication
D. Perform a lumbar puncture
Answer: A. Dim the lights in the child’s room.
Rationale:
In cases of suspected bacterial meningitis, it is important to provide a calming environment to
reduce stimulation and the risk of seizures, which are common in this condition. Dimming the
lights is a priority action to help minimize sensory input. While obtaining a CBC, administering
antipyretics, and performing a lumbar puncture are important actions, they are not the immediate
priority compared to ensuring the child's comfort and safety in a potentially seizure-prone state.
55. A nurse is caring for an infant who has rotavirus. Which of the following findings indicates
that the infant is moderately dehydrated?
A. Weight loss of 3%
B. Weight loss of 5%
C. Weight loss of 7%
D. Weight loss of 10%
Answer: C. Weight loss of 7%.
Rationale:
• Weight loss of 5-10% indicates varying degrees of dehydration in infants. A weight loss of 7%
typically corresponds to moderate dehydration. The following classifications are often used:
• Mild dehydration: Weight loss of up to 5%
• Moderate dehydration: Weight loss of 6-9%
• Severe dehydration: Weight loss of 10% or more
• Therefore, a 7% weight loss is indicative of moderate dehydration.

56. A nurse is providing teaching to the guardian of a school-age child who has seizure disorder.
Which of the following factors should the nurse include as a common trigger that increases the
risk of seizure?
A. High carbohydrate intake
B. Dehydration
C. Lack of sleep
D. Increased physical activity
Answer: C. Lack of sleep.
Rationale:
Lack of sleep is a well-documented trigger for seizures in children with seizure disorders. Other
common triggers can include stress, illness, missed medications, and certain environmental
factors. While dehydration and changes in diet can also impact seizure activity, lack of sleep is
particularly significant and often highlighted in patient education. It's essential for guardians to
understand these triggers to help manage and minimize the risk of seizures.
57. A nurse is reviewing the laboratory results of a preschooler who has gastroenteritis and notes
the client’s potassium level is 3.2 meq L which of the following assessment findings should the
nurse expect?
A. Oliguria
B. Hyperactivity
C. Increased thirst
D. Elevated heart rate
Answer: A. Oliguria.
Rationale:
A potassium level of 3.2 mEq/L indicates hypokalaemia, which can lead to various symptoms.
Oliguria, or decreased urine output, is a common finding in hypokalaemia due to its effects on
kidney function and fluid balance. Other symptoms might include muscle weakness, fatigue, and
cardiac dysrhythmias. In the context of gastroenteritis, where vomiting and diarrhoea can lead to
significant potassium loss, monitoring for oliguria is crucial for assessing the child's renal status
and electrolyte balance.

58. A nurse is planning care for an adolescent who has sickle cell anaemia. Which of the
following immunizations should the nurse include in the plan?
A. Measles, mumps, rubella (MMR)
B. Pneumococcal conjugate (PCV13)
C. Influenza
D. Varicella
Answer: B. Pneumococcal conjugate (PCV13).
Rationale:
Children and adolescents with sickle cell anaemia are at increased risk for infections, particularly
from encapsulated bacteria such as Streptococcus pneumoniae. The pneumococcal conjugate
vaccine (PCV13) is recommended to help protect against pneumococcal infections. In addition,
other vaccinations, including those for influenza and meningococcal disease, are also important,
but PCV13 specifically addresses a key risk for individuals with sickle cell anaemia.
59. A nurse is planning care for a child who has varicella. Which of the following interventions
should the nurse plan to include?
A. Administer aspirin for fever
B. Initiate airborne precautions
C. Encourage the child to take a warm bath
D. Isolate the child in a room with positive pressure ventilation
Answer: B. Initiate airborne precautions.
Rationale:
Varicella (chickenpox) is transmitted via airborne droplets and direct contact with the lesions.
Therefore, airborne precautions are necessary to prevent the spread of the virus to others. This
includes using an N95 respirator or equivalent mask when entering the room and ensuring the
room is equipped for airborne isolation. Other options, like administering aspirin, are
contraindicated in children with viral infections due to the risk of Reye's syndrome.
60. A nurse is planning care for a school-age child who has a new diagnosis of leg calve Perthes
disease. Which of the following interventions should the nurse include in the plan of care?
A. Encourage weight-bearing activities
B. Administer ibuprofen to the child for discomfort

C. Apply heat packs to the affected hip
D. Schedule the child for physical therapy sessions twice a week
Answer: B. Administer ibuprofen to the child for discomfort.
Rationale:
In children with Legg-Calve-Perthes disease, managing pain and inflammation is essential.
Ibuprofen is a nonsteroidal anti-inflammatory drug (NSAID) that can help alleviate discomfort.
Encouraging weight-bearing activities is not recommended as it may worsen the condition, and
heat packs are not typically indicated for this condition. Physical therapy may be helpful, but it
should be tailored to the child's specific needs and directed by a healthcare provider.
61. A nurse is caring for a two-year-old child who has cystic fibrosis and is being discharged from
the hospital. The nurse should ensure that which of the following pieces of equipment is available
for the child’s home?
A. Nebulizer machine
B. High-frequency chest compression vest
C. Pulse oximeter
D. Suction machine
Answer: B. High-frequency chest compression vest.
Rationale:
For a child with cystic fibrosis, a high-frequency chest compression vest is crucial for airway
clearance and promoting effective lung function. It helps to loosen and mobilize mucus from the
lungs. While a nebulizer and pulse oximeter can also be beneficial for managing cystic fibrosis,
the vest is specifically designed for respiratory therapy and is essential for daily care. A suction
machine may be used in some cases, but the vest is more directly related to the primary treatment
of cystic fibrosis.
62. A nurse is providing teaching for the parent of a child who has measles. Which of the
following information should the nurse include?
A. Use cold water to bathe the child
B. Bathe the child using tepid water
C. Apply hot water compresses to the child
D. Do not bathe the child at all

Answer: B. Bathe the child using tepid water.
Rationale:
When caring for a child with measles, tepid water baths can help reduce fever and provide
comfort. Tepid water (slightly warm) is recommended to avoid causing shivering, which can raise
body temperature. Cold water is not advised as it can lead to discomfort and shivering, and hot
water can further increase body temperature. Regular bathing is typically encouraged to maintain
hygiene, especially if the child is sweating or has a rash.
63. A nurse is assessing a toddler who has cystic fibrosis. Which of the following findings should
the nurse expect?
A. Diarrhoea
B. Steatorrhea
C. Constipation
D. Hematemesis
Answer: B. Steatorrhea.
Rationale:
In cystic fibrosis, the body has difficulty digesting fats due to pancreatic enzyme insufficiency,
leading to steatorrhea, which is characterized by bulky, fatty, and foul-smelling stools. This is a
common finding in toddlers with cystic fibrosis, as they may also experience issues with nutrient
absorption. Diarrhoea can occur, but steatorrhea specifically indicates fat malabsorption, making
it a key symptom to expect.
64. A nurse is planning to administer diphenhydramine 1.25 mg/kg IV to a school-age child who
weighs 55 lb. Available is diphenhydramine 50 mg0ml. How many mL should the nurse
administer. Round the answer to the nearest tenth. Use leading zeros that applies. Do not use a
trailing zero.
A. 0.4 mL
B. 0.5 mL
C. 0.6 mL
D. 0.7 mL
Answer: C. 0.6 mL
Rationale:

The calculated dosage of diphenhydramine for a school-age child weighing 55 lb (approximately
25 kg) at a rate of 1.25 mg/kg is 31.25 mg. Given the concentration of the medication is 50
mg/mL, the required volume to administer is 0.625 mL, which rounds to 0.6 mL.

NUMBER 2—4.7 stars
65. A nurse is providing education about dietary modifications to the parent of a school age child
who has glomerulonephritis. Which of the following information should the nurse include in the
teaching?
A. Increase the child's potassium intake
B. Decrease the child's sodium intake
C. Increase the child's protein intake
D. Provide high-sugar snacks
Answer: B. Decrease the child's sodium intake
Rationale:
In glomerulonephritis, reducing sodium intake helps manage hypertension and edema, which can
be associated with this condition. Limiting sodium can aid in reducing fluid retention and control
blood pressure, making it an important dietary modification for children with glomerulonephritis.
66. A nurse is providing teaching to the parents of a school-age child newly diagnosed with a
seizure disorder. The nurse should teach the parents to take which of the following actions during
a seizure?
A. Restrain the child to prevent movement
B. Place a spoon in the child's mouth
C. Clear the area of hard objects
D. Hold the child's head to keep it still
Answer: C. Clear the area of hard objects
Rationale:
During a seizure, it is essential to ensure the child's safety by clearing the area of hard or sharp
objects that could cause injury. Restraining the child or placing objects in their mouth can lead to

additional harm. The focus should be on preventing injury and allowing the seizure to occur
without interference.
67. A nurse is assessing an adolescent who has type 1 diabetes mellitus. Which of the following
findings is the nurse's priority?
A. HbA1C 11.5%
B. Recent weight loss
C. Elevated blood glucose levels
D. Reported episodes of hypoglycemia
Answer: A. HbA1C 11.5%
Rationale:
An HbA1C of 11.5% indicates poor long-term glycaemic control and significantly increases the
risk of complications related to diabetes. While other findings may also be concerning, the high
HbA1C reflects chronic hyperglycemia and requires immediate attention to adjust the diabetes
management plan to reduce the risk of long-term complications.
68. A nurse is providing anticipatory guidance to a parent of a 1- month-old infant. The nurse
should include that it is recommended to start this series of which of the following immunization
first?
A. Inactivated poliovirus (IPV)
B. Hepatitis B
C. Measles, mumps, rubella (MMR)
D. Diphtheria, tetanus, pertussis (DTaP)
Answer: B. Hepatitis B
Rationale:
The recommended immunization schedule indicates that the Hepatitis B vaccine should be
administered within the first month of life, ideally before the infant is discharged from the
hospital. The other vaccines listed typically start at later ages, with the inactivated poliovirus
vaccine starting at 2 months of age.
69. A nurse is reviewing the laboratory report of a toddler who has haemolytic uremic syndrome.
Which of the following findings should the nurse expect?

A. BUN 28 mg/dL
B. Haemoglobin 14 g/dL
C. Platelet count 300,000/mm³
D. Serum creatinine 0.5 mg/Dl
Answer: A. BUN 28 mg/dL
Rationale:
In haemolytic uremic syndrome (HUS), the kidneys are affected, leading to decreased renal
function. An elevated blood urea nitrogen (BUN) level, such as 28 mg/dL, indicates impaired
kidney function, which is consistent with HUS. Normal values for haemoglobin and platelet
count would be expected to be low in this condition, and serum creatinine would typically be
elevated rather than low.
70. A nurse is caring for a school-age child who is experiencing a sickle cell crisis. Which of the
following actions should the nurse take? (ATI pg. 126)
A. Apply warm compresses to the affected areas
B. Administer ibuprofen for pain relief
C. Encourage the child to exercise
D. Provide ice packs to the affected areas
Answer: A. Apply warm compresses to the affected areas
Rationale:
During a sickle cell crisis, applying warm compresses can help improve blood flow and reduce
pain in the affected areas. Cold can constrict blood vessels and exacerbate pain, so warm
compresses are more appropriate. Pain relief may involve medications like acetaminophen or
opioids, rather than ibuprofen, which is less effective for severe pain. Encouraging exercise is not
advisable during a crisis due to the increased risk of further complications.
71. A nurse is assessing a 6-month-old infant who has respiratory syncytial virus. The nurse
should immediately report which of the following finding to the provider?
A. Tachypnoea
B. Mild wheezing
C. Low-grade fever
D. Decreased appetite

Answer: A. Tachypnoea
Rationale:
Tachypnoea, or increased respiratory rate, is a concerning sign in an infant with respiratory
syncytial virus (RSV) as it can indicate worsening respiratory distress. It suggests that the infant
may be struggling to maintain adequate oxygenation and may require immediate medical
intervention. While mild wheezing and a low-grade fever can also be monitored, tachypnoea is a
more urgent finding that should be reported to the provider right away. Decreased appetite may
occur but is less critical compared to signs of respiratory distress.
72. A nurse is planning to teach an adolescent who is lactose intolerant about dietary guidelines.
Which of the following instructions should the nurse include in the teaching?
A. You can consume milk in small amounts to build tolerance.
B. You can replace milk with non-dairy sources of calcium.
C. You should avoid all dairy products completely.
D. You can take lactase enzyme supplements before consuming dairy.
Answer: B. You can replace milk with non-dairy sources of calcium.
Rationale:
For adolescents who are lactose intolerant, it is important to ensure they still receive adequate
calcium for bone health. Recommending non-dairy sources of calcium, such as fortified plantbased milks (like almond, soy, or oat milk), leafy green vegetables, and certain fish, helps them
maintain their nutritional needs without triggering their lactose intolerance. While some
individuals may tolerate small amounts of dairy or use lactase supplements, the focus on nondairy sources is a safer and more reliable option for managing lactose intolerance.
73. A nurse on a paediatric intensive care unit is caring for a toddler who weighs 12 kg (26.5 Ib)
and is postoperative following open heart surgery. Which of the following findings should the
nurse report to the provider?
A. Urine output of 15 mL in the last 2 hr
B. Heart rate of 120 beats per minute
C. Blood pressure of 85/50 mm Hg
D. Temperature of 37.5°C (99.5°F)
Answer: A. Urine output of 15 mL in the last 2 hr

Rationale:
In a postoperative paediatric patient, urine output is a critical indicator of kidney function and
overall fluid status. For a toddler weighing 12 kg, the expected urine output is at least 1 mL/kg/hr,
which equates to a minimum of 24 mL over the last 2 hours. A urine output of 15 mL in this time
frame indicates potential inadequate renal perfusion or fluid overload, which requires immediate
assessment and intervention. The other findings (heart rate, blood pressure, and temperature) are
within acceptable ranges and do not warrant immediate reporting.
74. A nurse is providing dietary teaching to a parent of a 10-month-old infant who has
phenylketonuria. Which of the following responses by the parent indicate an understanding of the
teaching?
A. "I will give her whole milk as a snack."
B. "I will steam carrots and cut them into small pieces for her."
C. "I will include cereals that are high in protein."
D. "I will prepare a peanut butter sandwich for her lunch."
Answer: B. "I will steam carrots and cut them into small pieces for her."
Rationale:
This response indicates understanding because vegetables like carrots are low in phenylalanine
and are suitable for a child with phenylketonuria (PKU). Steaming and cutting them into small
pieces also aligns with safe food preparation for an infant. The other options are inappropriate, as
they include high-protein foods that are not suitable for a child with PKU. Whole milk and highprotein cereals are sources of phenylalanine, which must be restricted in this condition. Peanut
butter is also high in protein and not appropriate for a PKU diet.
75. A nurse is providing teaching to the parent of a preschool-age child who has celiac disease.
Which of the following instructions should the nurse include?
A. "Your child can gradually reintroduce gluten after a few years."
B. "Your child will be on a gluten-free diet for the rest of her life."
C. "It's fine to give your child foods labeled as 'wheat-free'."
D. "Your child will outgrow celiac disease by the time she is a teenager."
Answer: B. "Your child will be on a gluten-free diet for the rest of her life."
Rationale:

Celiac disease is a lifelong autoimmune disorder where the ingestion of gluten leads to damage in
the small intestine. The only effective treatment is strict adherence to a gluten-free diet for life.
The other options are incorrect; gluten cannot be reintroduced, "wheat-free" foods may still
contain gluten, and celiac disease is not something that can be outgrown.
76. A nurse is administering albuterol by metered dose inhaler for a preschool-age child who is
experiencing an asthma exacerbation. Which of the following findings should the nurse report to
the provider?
A. Increased respiratory rate
B. Intercostal retractions
C. Mild wheezing
D. Use of accessory muscles
Answer: B. Intercostal retractions
Rationale:
Intercostal retractions indicate increased work of breathing and potential respiratory distress in a
preschool-age child experiencing an asthma exacerbation. This finding suggests that the child's
respiratory status may be deteriorating and requires further evaluation and intervention. While
increased respiratory rate, mild wheezing, and the use of accessory muscles are concerning,
intercostal retractions are particularly indicative of significant respiratory distress that should be
reported to the provider.
77. A nurse is caring for a school-age child who is 1 hr postoperative following it tonsillectomy.
Which of the following actions should the nurse take? (Select all that apply.)
A. Administer an analgesic to the child on a scheduled basis.
B. Observe the child for frequent swallowing.
C. Discourage the child from coughing.
D. Encourage the child to drink clear fluids.
E. Position the child on their back.
Answer: A. Administer an analgesic to the child on a scheduled basis.
B. Observe the child for frequent swallowing.
C. Discourage the child from coughing.
Rationale:

A. Administer an analgesic to the child on a scheduled basis. This is important for pain
management postoperatively, as adequate pain control can help the child recover more
comfortably.
B. Observe the child for frequent swallowing. Frequent swallowing can indicate bleeding and
should be monitored closely after tonsillectomy.
C. Discourage the child from coughing. Coughing can lead to increased throat irritation and
potential bleeding, so it should be discouraged.
Options D and E are not appropriate actions; encouraging clear fluids is important for hydration,
and the child should not be positioned flat on their back to avoid the risk of aspiration.
78. A nurse is caring for a school-age child who has heart failure. Which of the following findings
should the nurse expect? (select all that apply.)
A. Tachycardia
B. Dyspnea
C. Bounding peripheral pulses
D. Weight gain
E. Cool extremities
Answer: A. Tachycardia
B. Dyspnea
D. Weight gain
Rationale:
A. Tachycardia: Increased heart rate is a common compensatory mechanism in heart failure as the
body tries to maintain adequate cardiac output.
B. Dyspnea: Difficulty breathing or shortness of breath is typical due to fluid accumulation in the
lungs (pulmonary congestion).
D. Weight gain: Weight gain can occur due to fluid retention, which is common in heart failure.
C. Bounding peripheral pulses are generally not associated with heart failure; they might indicate
conditions like fever or hypervolemia.
E. Cool extremities might be seen in heart failure due to decreased perfusion, but are not as
characteristic as the other findings.

79. A nurse in an emergency department is assisting a toddler who has a head injury. Which of the
following findings should the nurse report to the provider?
A. Vomiting
B. Mild headache
C. Dizziness
D. Irritability
Answer: A. Vomiting
Rationale:
Vomiting after a head injury can be a sign of increased intracranial pressure or a more serious
brain injury, such as a concussion or bleeding. It is important to report this finding to the provider
for further evaluation and management. The other options, while they may be concerning, are less
immediately alarming than vomiting in the context of a head injury.
80. A nurse caring for a toddler who is in the terminal stage of neuroblastoma. The parents ask,
how can we help our child now? Which of the following responses by the nurse is appropriate?
A. "Stay close to your child."
B. "Let your child have some alone time."
C. "Focus on keeping your child comfortable."
D. "Encourage your child to be active."
Answer: A. "Stay close to your child."
Rationale:
In the terminal stage of illness, providing emotional support and presence is crucial. Staying close
to the child can offer comfort and reassurance, making the child feel loved and secure. While
comfort measures and allowing some alone time can also be important, the immediate response
emphasizes the significance of parental presence during this difficult time.
81. A nurse is preparing to administer cephalexin 25 mg/kg PO to a child who has otitis media
and weighs 22 kg (48.5 Ib). Available is Cephalexin solution 250 mg/5 mL how many mL should
the nurse administer? (Round to the nearest whole number. Using a leading Zero if applies. Do
not use a trailing zero.)
A. 9 mL
B. 11 mL

C. 13 mL
D. 15 mL
Answer: B. 11 mL
Rationale:
The correct dosage for the child weighing 22 kg is 550 mg, which corresponds to 11 mL of the
available cephalexin solution (250 mg/5 mL).
82. During a well-baby visit, the parent of a 2- week-old newborn tells the nurse, "My baby
always keeps her head tilt to the right side. The nurse should further assess which of the
following areas?
A. Cranial sutures
B. Sternocleidomastoid muscle
C. Cervical spine
D. Visual acuity
Answer: B. Sternocleidomastoid muscle
Rationale:
A persistent head tilt in a newborn can indicate a condition known as torticollis, which is often
related to tightness in the sternocleidomastoid muscle. Assessing this muscle can help determine
if it is contributing to the head tilt.
83. A nurse is caring for a single mother of a 6-month-old infant. During a well-baby visit, the
mother expresses feeling "inexperience" in caring for the baby. The nurse should recommend
which of the following community resources?
A. Parent enhancement center
B. Local library
C. Neighbourhood watch program
D. Emergency services
Answer: A. Parent enhancement center
Rationale:
A parent enhancement center typically offers support, education, and resources for new parents,
helping them develop their parenting skills and confidence in caring for their infants. This would
be a beneficial resource for the mother feeling inexperienced.

84. A nurse is admitting an infant who has GERD. Which of the following is the priority
assessment finding?
A. Wheezing
B. Vomiting
C. Poor weight gain
D. Irritability
Answer: A. Wheezing
Rationale:
Wheezing can indicate respiratory distress or complications from gastroesophageal reflux disease
(GERD), such as aspiration pneumonia. It is a critical finding that requires immediate attention
compared to other symptoms. While vomiting, poor weight gain, and irritability are also
concerning, wheezing poses a more immediate risk to the infant's respiratory status.
85. A nurse is caring for an infant who has severe dehydration. Which of the following clinical
findings should the nurse expect?
A. Rapid respirations
B. Warm, dry skin
C. Increased urinary output
D. Sluggish reflexes
Answer: A. Rapid respirations
Rationale:
In cases of severe dehydration, the body compensates for decreased blood volume and possible
shock by increasing respiratory rate (tachypnoea). Other findings may include dry mucous
membranes, decreased skin turgor, and lethargy or irritability. Warm, dry skin can also be present,
but increased urinary output would not be expected in dehydration.
86. A nurse is teaching a group of female adolescents about healthy eating. Which of the
following instructions should the nurse include in the teaching?
A. "Increase the amount of your dietary iron intake."
B. "Limit your intake of fruits and vegetables."
C. "Avoid all dairy products."

D. "Focus on high-sugar snacks for energy."
Answer: A. "Increase the amount of your dietary iron intake."
Rationale:
Adolescent females are at an increased risk for iron deficiency due to menstruation and higher
dietary needs. Encouraging an increase in dietary iron intake helps support overall health and
prevents anaemia. The other options are not appropriate for healthy eating guidelines.
87. A nurse is preparing to administer immunization to a 3-month-old infant. Which of the
following is an appropriate action for the nurse to take to deliver atraumatic care?
A. "Provide a pacifier coated with an oral sucrose solution prior to the injections."
B. "Administer the injection while the infant is lying flat."
C. "Perform the injection in the same arm for all immunizations."
D. "Encourage the parents to hold the infant tightly during the procedure."
Answer: A. "Provide a pacifier coated with an oral sucrose solution prior to the injections."
Rationale:
Offering a pacifier coated with an oral sucrose solution can help soothe the infant and reduce pain
perception during immunization, promoting a more positive experience. This aligns with the
principles of atraumatic care, which aim to minimize physical and emotional distress for
paediatric patients. The other options may not provide the same level of comfort or could increase
anxiety.
88. A nurse is caring for a child who has impetigo contagious that developed in the hospital.
Which of the following actions should the nurse take?
A. "Initiate contact isolation precautions."
B. "Administer systemic antibiotics."
C. "Apply topical corticosteroids."
D. "Encourage the child to share personal items to promote socialization."
Answer: A. "Initiate contact isolation precautions."
Rationale:
Impetigo contagious is a highly contagious skin infection caused by bacteria. Initiating contact
isolation precautions is essential to prevent the spread of the infection to other patients and staff.
While systemic antibiotics may be necessary for treatment, the first step to prevent transmission

is implementing proper isolation precautions. The other options do not adequately address
infection control.
89. A nurse is providing discharge teaching to the parents of a school-age child who has cystic
fibrosis. Which of the following responses by the parents indicate an understanding of the
teaching?
A. "I will give my child pancreatic enzymes with snacks and meals."
B. "I will limit my child's fluid intake to prevent dehydration."
C. "I will avoid high-calorie foods to manage weight."
D. "I will stop the chest physiotherapy once my child feels better."
Answer: A. "I will give my child pancreatic enzymes with snacks and meals."
Rationale:
Children with cystic fibrosis often have difficulty digesting food due to insufficient pancreatic
enzymes. Administering pancreatic enzymes with snacks and meals helps improve nutrient
absorption. The other statements indicate misunderstandings about the management of cystic
fibrosis, as fluid intake should be adequate, high-calorie foods are often encouraged to maintain
weight, and chest physiotherapy should be continued as prescribed, regardless of symptom
improvement.
90. A nurse is caring for a 4-year-old child who has meningitis and is receiving gentamicin.
Which of the following laboratory values should the nurse report to the provider?
A. BUN 15 mg/dL
B. Creatinine 1.4 mg/dL
C. Sodium 138 mEq/L
D. Potassium 4.2 mEq/L
Answer: B. Creatinine 1.4 mg/dL
Rationale:
Gentamicin is an aminoglycoside antibiotic that can be nephrotoxic. A creatinine level of 1.4
mg/dL is elevated for a child and may indicate impaired kidney function. It's important to report
this finding to the provider for further evaluation and potential adjustment of the medication
regimen. The other values are within normal ranges and do not require reporting.

91. A nurse is providing teaching to the parent of a school-age child who has ADHD and a new
prescription for methylphenidate. The nurse should explain that this medication will have which
of the following therapeutic effects?
A. Decreasing hyperactivity
B. Improving sleep patterns
C. Increasing focus
D. Reducing appetite
Answer: C. Increasing focus
Rationale:
Methylphenidate is a stimulant medication commonly used to treat ADHD. One of its primary
therapeutic effects is to increase focus and attention in children, helping them to concentrate
better on tasks. While it can also reduce hyperactivity, the main goal is to enhance the child's
ability to focus and complete tasks more effectively.
92. A nurse is teaching an adolescent how to manage his cystic fibrosis. which of the following
statements by the adolescent indicates an understanding of the teaching?
A. I will increase my intake of vitamin D
B. I will avoid physical activity to prevent lung infections
C. I will take my pancreatic enzymes only when I feel discomfort
D. I will skip treatments when I feel well
Answer: A. I will increase my intake of vitamin D
Rationale:
Adolescents with cystic fibrosis often require additional vitamins, particularly fat-soluble
vitamins like A, D, E, and K, due to malabsorption issues. Increasing vitamin D intake is essential
for bone health and overall well-being. The other options do not reflect appropriate management
strategies for cystic fibrosis.
93. A nurse in a provider's office is caring for a preschool-age child who might have acute
epiglottitis. Which of the following actions should the nurse take?
A. Provide humidified oxygen via nasal cannula
B. Encourage the child to lie supine
C. Obtain a throat culture for diagnosis

D. Administer ibuprofen for fever
Answer: A. Provide humidified oxygen via nasal cannula
Rationale:
In cases of suspected acute epiglottitis, ensuring adequate oxygenation is crucial, and providing
humidified oxygen can help keep the airway moist and reduce irritation. Immediate assessment
and monitoring are essential, as this condition can lead to airway obstruction. Options B, C, and
D are not appropriate initial actions; lying supine may worsen breathing difficulties, throat
cultures can provoke airway spasms, and ibuprofen does not address the immediate risks of
airway obstruction.
94. A nurse is providing teaching to the parents of a child who has impetigo. Which of the
following instructions should the nurse include in the teaching?
A. Apply bactericidal ointment to lesions
B. Keep the lesions covered with a bandage at all times
C. Encourage the child to scratch the lesions to promote healing
D. Share personal items, like towels, with the child to avoid transmission
Answer: A. Apply bactericidal ointment to lesions
Rationale:
Impetigo is a contagious bacterial skin infection that is typically treated with topical bactericidal
ointments, such as mupirocin, applied directly to the lesions. Keeping lesions covered may be
helpful, but the focus should be on applying the appropriate treatment. Scratching can lead to
further infection, and sharing personal items can increase the risk of spreading the infection.
Therefore, proper hygiene and treatment are key in managing impetigo effectively.
95. A nurse is preparing to perform a venipuncture to collect a blood sample from an infant.
Which of the following restraints should the nurse plan to use for this procedure?
A. Wrist restraint
B. Mummy restraint
C. Arm restraint
D. Leg restraint
Answer: B. Mummy restraint
Rationale:

The mummy restraint is commonly used for infants during procedures like venipuncture to help
keep them still and prevent sudden movements that could lead to injury. It securely wraps the
infant's arms and legs, allowing the nurse to have better access to the venipuncture site while
ensuring the infant's safety and comfort. Other types of restraints may not provide the same level
of control and may not be appropriate for this procedure.
96. A nurse is reviewing the laboratory report of a school age child who has rheumatic fever.
Which of the following laboratory findings should the nurse expect?
A. Decreased white blood cell count
B. Increased ant streptolysin O titter (ASO)
C. Decreased C-reactive protein (CRP)
D. Normal erythrocyte sedimentation rate (ESR)
Answer: B. Increased ant streptolysin O titter (ASO)
Rationale:
An increased ASO titter is indicative of a recent streptococcal infection, which is often the
precipitating factor for rheumatic fever. This finding helps confirm the diagnosis, as rheumatic
fever typically follows a streptococcal throat infection. Other laboratory findings such as elevated
C-reactive protein (CRP) and erythrocyte sedimentation rate (ESR) may also be present,
indicating inflammation, but the key finding associated with rheumatic fever is the elevated ASO
titter.
97. A nurses administering an opioid to an adolescent who is in sickle cell crisis. Which statement
is true regarding opioid pain management?
A. Oral opioid doses should be larger than parenteral doses.
B. Opioids can be safely administered for acute pain without fear of addiction.
C. Opioids should be avoided in patients with a history of substance use disorder.
D. All opioids have the same potency and can be interchanged without dose adjustments.
Answer: B. Opioids can be safely administered for acute pain without fear of addiction.
Rationale:
In acute pain situations, such as a sickle cell crisis, opioids are considered essential for effective
pain management. Concerns about addiction are generally secondary to the need for pain relief.
While it’s true that oral doses of opioids are often larger than parenteral doses due to lower

bioavailability, the key point here is that opioids are appropriate and safe for managing acute pain
crises, even in patients who may have risk factors for substance use disorder.
98. A nurse is planning care for an adolescent following repair of Meckel diverticulum. Which of
the following actions should the nurse include in the plan of care?
A. Maintain an NG tube for decompression.
B. Encourage early oral intake of solid foods.
C. Assess bowel sounds every 8 hours.
D. Administer oral antibiotics as prescribed.
Answer: A. Maintain an NG tube for decompression.
Rationale:
After surgical repair of Meckel diverticulum, maintaining an NG tube for decompression is
essential to prevent abdominal distention and facilitate recovery. It allows for the removal of
gastric secretions, reducing the risk of aspiration and promoting healing. Early oral intake of solid
foods is typically not encouraged until bowel function is confirmed, and bowel sounds are
assessed more frequently than every 8 hours. Antibiotics may be prescribed, but oral
administration is usually not initiated until the patient is ready for oral intake.
99. A nurse is preparing to perform peritoneal dialysis for a child who has an elevated serum
creatinine level. After explaining the procedure, which of the following action should the nurse
plan to take?
A. Obtain the child's weight.
B. Insert a urinary catheter.
C. Administer a sedative as prescribed.
D. Check the child's vital signs.
Answer: A. Obtain the child's weight.
Rationale:
Obtaining the child's weight is crucial before initiating peritoneal dialysis to assess the fluid status
and help determine the appropriate amount of dialysate to be used. Monitoring weight changes
helps evaluate the effectiveness of the dialysis treatment. While checking vital signs is also
important, the weight is a specific step in the preparation for dialysis. A urinary catheter is not
necessary for peritoneal dialysis, and sedatives are typically not required for the procedure.

100. A nurse is caring for an adolescent who is one hour postoperative following an
appendectomy. Which of the following findings should the nurse report to the provider?
A. Mild abdominal tenderness
B. Muscle rigidity
C. Slight fever (99.5°F or 37.5°C)
D. Clear, pale urine
Answer: B. Muscle rigidity
Rationale:
Muscle rigidity can indicate complications such as peritonitis or internal bleeding, which are
serious concerns following abdominal surgery like an appendectomy. This finding should be
reported to the provider promptly. Mild abdominal tenderness is common postoperatively, slight
fever may be expected as the body heals, and clear urine is generally not a cause for concern.
101. A nurse is caring for a preschool-age child who is postoperative following a tonsillectomy
and is clearing her throat frequently. Which of the following actions should the nurse take first?
A. Administer prescribed analgesics
B. Observe the child's throat with a flashlight
C. Offer cool fluids to the child
D. Document the finding in the child's chart
Answer: B. Observe the child's throat with a flashlight.
Rationale:
Frequent throat clearing in a child post-tonsillectomy may indicate bleeding, which can be a
serious complication. The first priority is to assess the situation by observing the throat for any
signs of bleeding before taking other actions. Administering analgesics or offering fluids can be
done afterward, depending on the assessment findings.
102. A nurse is planning care for a Toddler who has developed oral ulcers in response to
chemotherapy. Which of the following actions should the nurse include in the plan of care?
A. Clean the gums with saline-soaked gauze
B. Encourage the child to brush their teeth with a firm toothbrush
C. Provide spicy foods to stimulate appetite

D. Administer oral antibiotics regularly
Answer: A. Clean the gums with saline-soaked gauze.
Rationale:
Cleaning the gums with saline-soaked gauze helps maintain oral hygiene and can provide some
relief for the oral ulcers caused by chemotherapy. It’s important to avoid harsh or abrasive
cleaning methods and to refrain from irritating foods during this time.
103. A nurse is planning care for a child immediately following the insertion of a chest tube for
continuous suction with a closed drainage system. Which of the following interventions should
the nurse include in the plan of care?
A. Change the chest tube insertion site dressing every 12 hr.
B. Monitor the child’s vital signs every 8 hr.
C. Clamp the chest tube periodically to assess lung re-expansion.
D. Notify the provider if the drainage is less than 30 mL/hr.
Answer: A. Change the chest tube insertion site dressing every 12 hr.
Rationale:
Regular dressing changes at the insertion site are essential to prevent infection and maintain a
sterile environment. The dressing should typically be changed based on facility policy, but every
12 hours is a common interval. Monitoring vital signs more frequently (usually every 1-2 hours)
and assessing drainage output is also important, but changing the dressing is critical for infection
prevention. Clamping the tube and specific output thresholds should be performed based on
clinical guidelines and the provider's orders.
104. A nurse is prioritizing care for 4 clients. Which of the following clients should the nurse
assess 1st?
A. An adolescent who has sickle cell anaemia and slurred speech.
B. A child with asthma who is wheezing.
C. A toddler with a high fever and rash.
D. An infant with dehydration due to diarrhoea.
Answer: A. An adolescent who has sickle cell anaemia and slurred speech.
Rationale:

The adolescent with sickle cell anaemia and slurred speech is a priority because slurred speech
may indicate a potential stroke, a critical and time-sensitive condition requiring immediate
assessment and intervention. While the other clients also require care, the potential for a stroke
makes this situation the highest priority.
105. A nurse is assisting an adolescent who has Cushing's syndrome. Which of the following
findings should the nurse expect?
A. Blood glucose 320 mg/dL
B. Low serum potassium level
C. Weight loss
D. Decreased cortisol levels
Answer: A. Blood glucose 320 mg/dL
Rationale:
In Cushing's syndrome, elevated cortisol levels can lead to insulin resistance and increased blood
glucose levels. A blood glucose level of 320 mg/dL is significantly high and consistent with
hyperglycemia seen in this condition. The other options are not typical findings associated with
Cushing's syndrome; patients often experience weight gain, not loss, and potassium levels may be
low due to the effects of cortisol.
106. A nurse is caring for a preschooler who has a brain tumor. Which of the following findings is
the priority for the nurse to report to the provider?
A. Diplopia
B. Headache
C. Nausea
D. Changes in behavior
Answer: A. Diplopia
Rationale:
Diplopia (double vision) can indicate increased intracranial pressure or involvement of cranial
nerves, which may suggest a more severe progression of the tumor or complications. While
headaches, nausea, and behavioral changes are also concerning, the presence of diplopia is
particularly critical and should be reported immediately to the provider for further evaluation and
intervention.

107. A charge nurse is planning care for an infant who has failure to thrive. which of the
following actions should the nurse include in the plan of care?
A. Assign consistent nursing staff to care for the infant
B. Allow the parents to visit only during specific hours
C. Encourage the use of formula instead of breast milk
D. Provide a strict feeding schedule with minimal flexibility
Answer: A. Assign consistent nursing staff to care for the infant
Rationale:
Consistent nursing staff helps establish a trusting relationship with the infant and can lead to
better assessment and intervention for their needs. This continuity can also support the parents
and promote a more stable environment for the infant, which is essential in managing failure to
thrive. The other options may not effectively support the infant’s emotional or nutritional needs.
108. A nurse is providing discharge teaching to the parents of an infant who is at risk for sudden
infant death syndrome is (SIDS). Which of the following statements by the parents indicates an
understanding of the teaching?
A. "I will place my baby on their stomach to sleep."
B. "I will use a soft blanket and stuffed animals in the crib."
C. "I will dress my baby in lightweight clothing to sleep."
D. "I will let my baby sleep in a car seat for naps."
Answer: C. "I will dress my baby in lightweight clothing to sleep."
Rationale:
Dressing the baby in lightweight clothing helps prevent overheating, which is a risk factor for
SIDS. It is crucial to ensure that the baby sleeps on their back in a safe sleep environment, free
from soft bedding and toys. The other statements reflect unsafe practices that could increase the
risk of SIDS.
109. A nurse is caring for a child who has acute glomerulonephritis. Which of the following
findings should the nurse expect?
A. Increased urine output
B. Periorbital edema

C. Hyperglycemia
D. Decreased blood pressure
Answer: B. Periorbital edema
Rationale:
Periorbital edema is a common finding in acute glomerulonephritis due to fluid retention and
inflammation. Other typical findings may include hypertension and oliguria (decreased urine
output), but increased urine output and hyperglycemia are not associated with this condition.
110. A nurse is assessing a 1-month- old infant at a well-child visit. Identify the location the nurse
should stroke to elicit this rooting reflex. (You will find hot spot to select in the artwork below.
Select only the hot spot that corresponds to your answer. )
A. Forehead
B. Cheek
C. Chin
D. Neck
Answer: B. Cheek
Rationale:
The rooting reflex is elicited when the infant's cheek is stroked. This reflex helps the infant turn
their head toward the stimulus, which is an important behavior for breastfeeding. Stimulating the
cheek encourages the baby to search for food, demonstrating the instinctual nature of feeding in
newborns.
111. A nurse is providing postoperative care for a child following an arterial cardiac
catheterization. Which of the following actions should the nurse take?
A. Flex the affected extremity every hour.
B. Keep the affected extremity straight for at least 6 hr.
C. Apply a warm compress to the catheterization site.
D. Encourage the child to ambulate within 2 hours post-procedure.
Answer: B. Keep the affected extremity straight for at least 6 hr.
Rationale:
After an arterial cardiac catheterization, it is crucial to keep the affected extremity straight for a
minimum of 6 hours to prevent complications such as bleeding or hematoma formation at the

insertion site. Flexing the extremity or applying heat can increase the risk of complications.
Encouraging ambulation too soon may also pose a risk, so rest and monitoring are essential
during the initial recovery period.
112. A nurse in a provider's office is providing teaching to the parents of a preschooler who has
Down syndrome. Which of the following statements by one of the parents indicate an
understanding of the instructions?
A. "We'll expect our son to learn new skills quickly without assistance."
B. "We won't worry about his progress; he’ll catch up eventually."
C. "We'll be sure to demonstrate a new skill before expecting our son to perform it."
D. "We'll allow him to learn at his own pace, without any guidance."
Answer: C. "We'll be sure to demonstrate a new skill before expecting our son to perform it."
Rationale:
This statement indicates an understanding of the importance of modeling and direct instruction
for children with Down syndrome. Demonstrating a new skill helps reinforce learning and
provides a clear example for the child to follow. Children with Down syndrome often benefit
from visual and practical demonstrations, making this approach effective in supporting their
learning and development.
113. A nurse is teaching a parent of a 10-month-old infant about home safety. Which of the
following instructions should the nurse include in the teaching? (Select all that apply.) Place gates
at the top and bottom of the stairs.
A. Place gates at the top and bottom of the stairs.
B. Ensure the crib mattress is in the lowest position.
C. Use a harness when placing the infant in a high chair.
D. Keep small objects that could be choking hazards out of reach.
E. Allow the infant to explore cabinets freely.
Answer: A. Place gates at the top and bottom of the stairs.
B. Ensure the crib mattress is in the lowest position.
C. Use a harness when placing the infant in a high chair.
D. Keep small objects that could be choking hazards out of reach.
Rationale:

Place gates at the top and bottom of the stairs: This prevents the infant from falling down the
stairs, which is a significant safety hazard.
Ensure the crib mattress is in the lowest position: Lowering the crib mattress reduces the risk of
the infant climbing or falling out, providing a safer sleeping environment.
Use a harness when placing the infant in a high chair: This helps to secure the infant and prevents
falls, promoting safety during feeding.
Keep small objects that could be choking hazards out of reach: This is crucial to prevent choking,
as infants tend to explore their environment by putting objects in their mouths.
114. A nurse is providing discharge teaching to a parent of a toddler who has a
ventriculoperitoneal shunt. which of the following statements by the parents indicates an
understanding of the teaching?
A. "I should call my doctor if my child begins vomiting."
B. "I will give my child extra fluids if they have a fever."
C. "It's normal for my child to have a bulging fontanelle."
D. "My child can return to all normal activities immediately."
Answer: A. "I should call my doctor if my child begins vomiting."
Rationale:
"I should call my doctor if my child begins vomiting.": This statement indicates understanding, as
vomiting can be a sign of increased intracranial pressure or shunt malfunction in a child with a
ventriculoperitoneal shunt. It's crucial for parents to monitor their child for symptoms that may
indicate complications and to seek medical advice promptly.
115. A nurse in a provider's office is assessing the vital signs of a 2-year-old child at a well-child
visit. Which of the following findings should the nurse report to the provider?
A. Blood pressure 118/74 mm Hg
B. Heart rate 110 bpm
C. Respiratory rate 24 breaths/min
D. Temperature 99°F (37.2°C)
Answer: A. Blood pressure 118/74 mm Hg
Rationale:

A blood pressure of 118/74 mm Hg in a 2-year-old is considered elevated. Normal blood pressure
for this age group typically ranges from about 90/50 mm Hg to 110/70 mm Hg. It's important for
the nurse to report this finding to the provider for further evaluation, as elevated blood pressure in
young children can indicate underlying health issues that need to be addressed.
116. A nurse is assessing a 3-month-old infant who has diarrhoea. Which of the following
findings should the nurse expect?
A. Decreased urine output
B. Increased haematocrit
C. Decreased skin turgor
D. Elevated blood pressure
Answer: B. Increased haematocrit
Rationale:
In a 3-month-old infant with diarrhoea, dehydration is a common concern. When dehydration
occurs, there is often a relative increase in haematocrit due to the loss of fluid volume, leading to
a higher concentration of red blood cells. This finding is consistent with dehydration.
117. A nurse is preparing to administer imipenem/cilastatin 25 mg/kg to a child who weighs 77
Ib. How many mg should the nurse plan to administer? (Round the answer to the nearest whole
number. Use a leading zero if it applies. Do not use a trailing zero.)
A. 625 mg
B. 750 mg
C. 875 mg
D. 1000 mg
Answer: C. 875 mg
Rationale:
To calculate the dosage, the child's weight in pounds (77 lbs) is converted to kilograms
(approximately 35 kg). Then, multiplying by the prescribed dosage (25 mg/kg) yields 875 mg.
This ensures the child receives the correct medication dosage based on their weight.
118. A nurse is providing teaching to a parent of an infant who has a 1 cm (0.4 in) umbilical
hernia. Which of the following instructions should the nurse include in the teaching?

A. "The bulge can temporarily enlarge when your baby cries."
B. "Surgery is always necessary for a hernia of this size."
C. "You should avoid allowing your baby to cry."
D. "The hernia will disappear by the time your baby is 3 months old."
Answer: A. "The bulge can temporarily enlarge when your baby cries."
Rationale:
This statement is correct because umbilical hernias can appear more prominent when the infant is
crying or straining. Parents should be informed that this is a normal occurrence and not a cause
for immediate concern. While most umbilical hernias in infants are benign and often resolve on
their own, the other options may mislead parents about the necessity of surgery or the nature of
the hernia.
119. A nurse is admitting a child who has pertussis. Which of the following transmission-based
precautions should the nurse initiate?
A. Airborne
B. Contact
C. Droplet
D. Standard
Answer: C. Droplet
Rationale:
Pertussis (whooping cough) is primarily transmitted through respiratory droplets when an
infected person coughs or sneezes. Therefore, droplet precautions should be initiated to prevent
the spread of the infection to others. This includes wearing a mask when in close contact with the
patient and ensuring the patient wears a mask when being transported outside of their room.
120. A nurse is assessing a toddler who has a history of lead poisoning. Which of the following
actions should the nurse take?
A. Perform development testing for delays
B. Order a blood lead level test
C. Initiate chelation therapy
D. Assess for gastrointestinal symptoms
Answer: A. Perform development testing for delays

Rationale:
Children with a history of lead poisoning are at risk for developmental delays, so it's important
for the nurse to perform developmental assessments to identify any potential issues early. While
other options may be relevant in different contexts (like checking lead levels or assessing
symptoms), the immediate action related to developmental concerns is to perform testing for
delays.
121. A nurse is reviewing the medical record of a 24-month-old child who has acute lymphocytic
leukaemia. Which of the following actions should the nurse take? (Click on the Exhibit button for
additional information about the client. There are three tabs that contain separate categories of
data.)
A. Initiate bleeding precautions.
B. Administer a routine immunization schedule.
C. Start a high-protein diet.
D. Provide regular developmental screenings.
Answer: A. Initiate bleeding precautions.
Rationale:
Initiating bleeding precautions is critical for a child with acute lymphocytic leukaemia due to the
risk of thrombocytopenia and potential bleeding complications. This includes measures to
minimize injury and monitor for signs of bleeding. The other options, while important in different
contexts, do not address the immediate concerns related to the child's condition.
122. A school nurse is assessing a 7-year-old student. The nurse should identify which of the
following findings as a potential indicator of physical abuse?
A. Bruising around the wrists.
B. Frequent stomach-aches.
C. Sudden changes in academic performance.
D. Complaints of being tired.
Answer: A. Bruising around the wrists.
Rationale:
Bruising around the wrists can be a potential indicator of physical abuse, especially if the bruising
is in a pattern consistent with restraint or being held. While the other options may also raise

concerns for various issues, they are not specific indicators of physical abuse. It's essential for the
nurse to conduct a thorough assessment and follow up on any suspicions of abuse.
123. A nurse is providing teaching to the parent of a school-age child who has diabetes mellitus
about managing diabetes during illness. Which of the following statements by the parent indicate
an understanding of the teaching?
A. "I will increase the amount of fluids I offer my child."
B. "I will reduce my child's insulin dosage during illness."
C. "I should avoid checking my child's blood glucose levels."
D. "I can let my child skip meals if they don’t feel well."
Answer: A. "I will increase the amount of fluids I offer my child."
Rationale:
During illness, children with diabetes need to stay hydrated, as illness can increase the risk of
dehydration and affect blood glucose levels. Increasing fluid intake helps maintain hydration and
can aid in managing blood glucose levels. The other statements indicate misunderstandings about
diabetes management during illness; insulin dosages may need adjustments, blood glucose levels
should be monitored, and meals should not be skipped without consulting a healthcare provider.
124. A nurse is providing discharge teaching to the parents of a toddler who has iron deficiency
anaemia and new prescription for ferrous sulfate elixir. Which of the following instructions
should the nurse include?
A. "Administer this medication to your child with a dropper."
B. "Give this medication with milk to improve absorption."
C. "Ensure your child takes this medication on an empty stomach."
D. "Store the medication in a warm place."
Answer: C. "Ensure your child takes this medication on an empty stomach."
Rationale:
Ferrous sulfate should ideally be administered on an empty stomach to enhance absorption. It is
important to avoid giving it with milk or antacids, as these can inhibit iron absorption. Using a
dropper is acceptable, but the primary focus should be on proper timing with respect to meals for
optimal effectiveness. Storing the medication in a cool, dry place is recommended rather than a
warm place.

125. A nurse is caring for an infant who has tetralogy of Fallot and is having a hyper cyanotic
episode after crying. Which of the following interventions should the nurse implement?
A. "Administer oxygen via nasal cannula."
B. "Place the infant in the knee-chest position."
C. "Give the infant a dose of morphine."
D. "Start an IV of normal saline."
Answer: B. "Place the infant in the knee-chest position."
Rationale:
During a hyper cyanotic episode (also known as a "tet spell") in an infant with tetralogy of Fallot,
placing the infant in the knee-chest position helps reduce the right-to-left shunt by increasing
systemic vascular resistance. This can improve oxygenation and decrease cyanosis. While oxygen
may also be administered, the immediate action to alleviate the episode is the knee-chest position.
Other interventions like IV fluids or morphine might be necessary but are not the priority in this
acute situation.
126. A nurse is providing teaching to an adolescent who has Vulvovaginitis. Which of the
following statements should the nurse include in the teaching?
A. "You should use scented products for hygiene."
B. "Apply a warm, moist compress three times per day."
C. "It's best to avoid all physical activities."
D. "Wear tight-fitting underwear to keep the area dry."
Answer: B. "Apply a warm, moist compress three times per day."
Rationale:
Applying a warm, moist compress can help soothe irritation and reduce discomfort associated
with vulvovaginitis. It is important to maintain hygiene without using scented products or
irritants, and wearing breathable cotton underwear is usually recommended instead of tight-fitting
options. Avoiding all physical activities is unnecessary; gentle activities can usually be continued
unless otherwise advised by a healthcare provider.

127. A nurse is providing discharge instructions to the parents of a toddler who has heart failure
and a new prescription for digoxin. Which of the following statements indicate an understanding
of the instructions?
A. "I will monitor my child's heart rate before each dose."
B. "It's okay to give the medication even if my child is vomiting."
C. "I can give the medication with any food or drink."
D. "If my child misses a dose, I can give two doses at the next scheduled time."
Answer: A. "I will monitor my child's heart rate before each dose."
Rationale:
Monitoring the heart rate before administering digoxin is crucial, as digoxin can affect heart
rhythm. Parents should be instructed to hold the medication if the heart rate is below a certain
threshold (often 90-110 bpm in toddlers, but specific guidance should be provided by the
healthcare provider). Vomiting, taking it with food or drink, and doubling doses are not
appropriate actions and could lead to complications or toxicity.
128. A nurse is planning on in-service for parents of school- age children about the treatment of
pediculosis capitis. Which of the following instructions should the nurse plan to include in the
teaching?
A. "Apply a topical insecticide to the scalp and hair."
B. "Use a regular comb to remove nits."
C. "Wash all clothing and bedding in hot water."
D. "Remove nits from the child's hair using a fine-toothed comb."
Answer: D. "Remove nits from the child's hair using a fine-toothed comb."
Rationale:
While applying a topical insecticide is important, thorough removal of nits (lice eggs) is essential
to prevent reinfestation. A fine-toothed comb is the most effective tool for this task. Regular
combs may not be effective at removing nits. Additionally, washing clothing and bedding in hot
water is recommended, but it should be noted that removing nits is a crucial step in managing
pediculosis capitis.
129. A nurse is assessing an adolescent who has infectious mononucleosis. Which of the
following findings should the nurse expect?

A. Jaundice
B. Cervical adenopathy
C. Hepatomegaly
D. Rash
Answer: B. Cervical adenopathy
Rationale:
Cervical adenopathy (swollen lymph nodes in the neck) is a common finding in adolescents with
infectious mononucleosis, which is caused by the Epstein-Barr virus. Other typical findings may
include fatigue, sore throat, and fever, but cervical adenopathy is a hallmark symptom. Jaundice
and hepatomegaly can occur but are less common, and a rash is not a characteristic feature of
mononucleosis.
130. A nurse in an emergency department is assisting an adolescent who reports inhalation of
gasoline. Which of the following findings should the nurse expect?
A. Euphoria
B. Ataxia
C. Bradycardia
D. Hypertension
Answer: B. Ataxia
Rationale:
Inhalation of gasoline can lead to neurological symptoms, including ataxia, which is
characterized by lack of coordination and balance. Other potential symptoms of gasoline
inhalation may include confusion, dizziness, or even respiratory distress. Euphoria is also a
possibility but ataxia is a more significant and concerning neurological sign that the nurse would
expect. Bradycardia and hypertension are less commonly associated with gasoline inhalation.
131. A nurse is preparing to assess a 4-year-old child's visual acuity. Which of the following
actions should the nurse plan to take?
A. Use a Snellen chart
B. Use a Tumbling E chart
C. Use a Landolt C chart
D. Use a picture chart

Answer: B. Use a Tumbling E chart
Rationale:
The Tumbling E chart is appropriate for assessing visual acuity in young children who may not be
able to read letters or identify pictures. It allows children to indicate the direction of the letter "E"
without needing to know the alphabet, making it a suitable choice for a 4-year-old child. The
Snellen chart and picture charts may not be as effective for this age group.
132. A nurse in an emergency department is caring for a child following an overdose of
acetylsalicylic acid. Which of the following medications should the nurse plan to administer?
A. Activated charcoal
B. N-acetylcysteine
C. Phytonadione (Vitamin K)
D. Sodium bicarbonate
Answer: D. Sodium bicarbonate
Rationale:
In cases of acetylsalicylic acid (aspirin) overdose, sodium bicarbonate is often administered to
help alkalinize the urine, which can enhance the elimination of salicylates from the body.
Phytonadione (Vitamin K) is not indicated for aspirin overdose; it is primarily used for
anticoagulant reversal. Activated charcoal may be used if the overdose is recent, but sodium
bicarbonate is the specific treatment to address the effects of salicylate toxicity.
133. A nurse is providing teaching to the parents of a toddler who is exhibiting negativism during
mealtimes. Which of the following statements by the nurse is appropriate?
A. "You should insist that she eats everything on her plate."
B. "Let her choose between two healthy options."
C. "Avoid giving her any choices at mealtimes."
D. "Make mealtimes more structured with strict rules."
Answer: B. "Let her choose between two healthy options."
Rationale:
Toddlers often exhibit negativism as a way to assert their independence. Offering choices, such as
between two healthy options, empowers the child and can reduce power struggles. This approach

fosters a positive mealtime experience and encourages healthy eating habits while respecting the
child's autonomy.
134. A nurse in an emergency department is caring for a child who weighs 18 kg (39.7 Ib) and
ingested six 500 mg acetaminophen tablets 4 hr ago. Which of the following actions should the
nurse take?
A. Prepare to give oral N-acetylcysteine (NAC).
B. Administer activated charcoal.
C. Perform a gastric lavage.
D. Consult with a poison control center.
Answer: A. Prepare to give oral N-acetylcysteine (NAC).
Rationale:
The child ingested six 500 mg acetaminophen tablets, totaling 3000 mg (3 g). Given the child's
weight of 18 kg, the calculated dose is approximately 166.7 mg/kg (3000 mg/18 kg). This
exceeds the toxic threshold of 150 mg/kg, indicating a potential for liver damage.

EXTRAS
135. Teaching the parents of a school-aged child who has a new diagnosis of osteomyelitis of the
tibia. The nurse should identify that which of the following statements by the parents indicates an
understanding of the teaching?
A. "My child will receive antibiotics for several weeks."
B. "My child will need to stay in the hospital for the entire treatment."
C. "We can stop the antibiotics as soon as my child feels better."
D. "My child will need regular follow-up appointments to monitor the treatment."
Answer: A. "My child will receive antibiotics for several weeks."
Rationale:
This statement shows an understanding that osteomyelitis typically requires a prolonged course of
antibiotics, often several weeks to ensure complete resolution of the infection.

136. A nurse is auscultating the lungs of an adolescent who has asthma. The nurse should identify
the sound as which of the following? Click the audio button to listen.
A. Wheezing
B. Rhonchi
C. Stridor
D. Tachypnoea
Answer: A. Wheezing
Rationale:
Wheezing is a high-pitched, whistling sound that occurs due to narrowed airways and is
commonly associated with asthma. It is typically heard during expiration but can also be present
during inspiration in severe cases.
137. A nurse in an emergency department is caring for a school-age child who is experiencing an
anaphylactic reaction. Which of the following is the priority action by the nurse?
A. Administer IM epinephrine to the child.
B. Place the child in a supine position.
C. Start an intravenous (IV) line for fluid resuscitation.
D. Administer antihistamines to the child.
Answer: A. Administer IM epinephrine to the child.
Rationale:
Administering IM epinephrine is the most critical and immediate action in anaphylaxis.
Epinephrine rapidly counteracts the severe allergic reaction by constricting blood vessels,
relaxing airway muscles, and reducing swelling. Immediate administration can be lifesaving.
138. The nurse is preparing to administer an to a . Which of the following actions should the
nurse plan to take?
A. Administer the immunization using a 24-gauge needle.
B. Administer the immunization using a 22-gauge needle.
C. Administer the immunization intramuscularly in the deltoid muscle.
D. Administer the immunization subcutaneously in the thigh.
Answer: C. Administer the immunization intramuscularly in the deltoid muscle. (assuming the
patient is old enough)

Rationale:
A 24-gauge needle can be used for subcutaneous injections, but it is generally not appropriate for
intramuscular (IM) vaccinations, which require a larger gauge needle to ensure proper delivery of
the vaccine.
139. A nurse is reviewing the of an infant who is receiving treatment for. The nurse should
identify which of the following laboratory values indicates of the current treatment?
A. Sodium 130 mEq/L
B. Sodium 140 mEq/L
C. Sodium 150 mEq/L
D. Sodium 135 mEq/L
Answer: B. Sodium 140 mEq/L
Rationale:
This value indicates hyponatremia (low sodium), which would suggest that the treatment is not
effective if the infant is being treated for low sodium levels.
140. The nurse is providing teaching about to the parents of a preschooler. Which of the following
play activities should the nurse recommend for the child?
A. Playing dress-up
B. Watching television
C. Playing video games
D. Building with small LEGO bricks
Answer: A. Playing dress-up
Rationale:
This activity encourages imagination and creativity, allows for role-playing, and helps develop
social skills and language as children enact different characters and scenarios. It promotes
cognitive and emotional development.
141. A nurse is teaching the parents of a newborn about ways to prevent sudden infant death
syndrome SIDS. Which of the following instructions should the nurse include?
A. Give the infant a pacifier at bedtime.
B. Place the infant on their stomach to sleep.

C. Use soft bedding, such as blankets and pillows, in the crib.
D. Feed the infant right before sleep to ensure they stay asleep.
Answer: A. Give the infant a pacifier at bedtime.
Rationale:
Research suggests that using a pacifier during naps and bedtime can reduce the risk of SIDS. The
pacifier may help keep the airway open and prevent the infant from rolling into a prone position.
142. A nurse is assessing an infant who has pneumonia. Which of the following findings is the
priority for the nurse to report to the provider?
A. Nasal flaring
B. Mild cough
C. Low-grade fever
D. Decreased appetite
Answer: A. Nasal flaring
Rationale:
Nasal flaring is a sign of respiratory distress and indicates that the infant is struggling to breathe.
It is a priority finding that requires immediate reporting and intervention, as it may suggest
hypoxia or increased work of breathing.
143. A school nurse is assessing a school-age child blood pressure while he is seated in a chair.
The child starts to experience a tonic-colonic seizure. Which of the following actions should the
nurse take first?
A. Assist the child to a side-lying position on the floor.
B. Place a padded tongue blade in the child's mouth.
C. Call for emergency assistance immediately.
D. Move any nearby objects out of the way.
Answer: A. Assist the child to a side-lying position on the floor.
Rationale:
This is the priority action. Positioning the child on their side helps to maintain an open airway
and allows any secretions to drain, reducing the risk of aspiration. It also prevents injury during
the seizure.

Document Details

Related Documents

person
Ethan Williams View profile
Close

Send listing report

highlight_off

You already reported this listing

The report is private and won't be shared with the owner

rotate_right
Close
rotate_right
Close

Send Message

image
Close

My favorites

image
Close

Application Form

image
Notifications visibility rotate_right Clear all Close close
image
image
arrow_left
arrow_right